Text Completion

Pataasin ang iyong marka sa homework at exams ngayon gamit ang Quizwiz!

Before his marriage the Duke had led an austere existence and now regarded the affectionate, somewhat (i)____ behavior of his young wife as simply (ii)____ . Blank (i) A. restrained B. frivolous C. ungainly Blank (ii) D. delightful E. sublime F. puerile

*I missed this* -Here I thought the contrast was between the Duke's old self and the new self and thus we would we looking for an opposite to the word "ascetic". That's why I went with D for the second blank -But the second blank is "puerile" which means "childishly silly and trivial", which makes sense -Before he would be prone to strict but now thinks the behavior is just trivial and silly

The success of the business venture ____ his expectations; he never thought that the firm would prosper. A. confirmed B. belied C. nullified D. fulfilled E. ratified

*I missed this* -Here I narrowed down to B and C and picked C which is wrong because nullify means to "cancel legally" and that doesn't make sense. I thought B couldn't be it because it applies only to feelings but that is wrong logic... *When you come across two words of which you know one even thought it is not the word that you came up with you really have to eliminate on that basis of asking "why is this wrong" and unless you can come up with a good reason, this is the right answer.* For ex: does belie work? Yes, because something is the case and it is opposite to what we believe or the reality is...

Jacques was a born ____________ : he would often regale those around him with stories from his fascinating childhood. A) dissembler B) iconoclast C) raconteur D) sentimentalist E) maverick

-Not challenging but could become challenging if the context changes a bit. For example here we just need a word for a "good story teller". But what if Jacques was someone who told stories about his past and was overly mawkish about them? Then it would be sentimental because that is the main point

The Hellenistic and Judaic philosophy of the early centuries did not so much ____________ ancient Greek philosophy as it did ____________ the Platonic concepts of this time with its understanding of the way in which an ideal world, or one of perfect forms, ____________ the existence of a perfect being. Even the philosophy of the Middle Ages was so inextricably bound with the ideas of ancient Greece that many philosophers could hardly imagine discussing the existence of a perfect being without invoking the conceptual framework laid down by Plato more than a thousand years earlier. Blank (i) A) adapt B) displace C) foreshadow Blank (ii) D) supplant E) reconcile F) corrupt Blank (iii) G) allowed for H) circumvented I) called into question

* I missed this because of a misinterpretation* -I did the first blank after figuring out some sort of a meaning and went with B and based off that I picked E for the second blank and that gave me the sense that these guys aren't trying to "displace" but "reconcile" and because reconcile would mean trying to "connect two unrelated things" I thought I would work best but that's not the case. -The last part says that "the Greek templates" were relied upon to understand things about "the perfect being" so the Hellenistic and Judaic are using it to reconcile and see how it allowed for the perfect being -Notice nothing in the TC says that the people went against or called into question but the last par tis largely positive. Based off this you have to make your choice. G is the answer you go for because at least it is an explicit clue even though it is not the one you came up with

Stress-induced amnesia is a rare and (i)____ phenomenon; it strikes the patient apparently without warning and the memory loss can be as (i)____ as that induced by (iii)____ trauma. Blank (i) A. devastating B. venial C. pervasive Blank (ii) D. generic E. limited F. complete Blank (iii) G. unexpected H. mental I. physical

*I missed this - idiomatic phrase/syntax* *The words have to connect and make sense as a whole* -The first two blanks are easy -"as that induced by ______ trauma" means that we are looking for something opposite to mental "'stress-induced"

The publishers, unwilling to (i)____ the entire risk, insisted that the author pay half the cost of the initial print run of his (ii)____ new book. Blank (i) A. hedge B. shoulder C. mitigate Blank (ii) D. unexceptionable E. controversial F. jaundiced

*I missed this - tricky word that means something different* -I thought unexceptional meant "not great/mediocre" but it means "you can't raise objections to it" thus E works better

*Rely on your knowledge of words and meanings. At this point you are well-equipped not to look anything up. Besides it goes against preparing you for the effing test.* Do not be fooled by her (i)____ manner; her superficial (ii)____ belies her worldliness. Blank (i) A. unsophisticated B. gregarious C. off-hand Blank (ii) D. proficiency E. naiveté F. seriousness

*I missed this* -At first I had the right answer A and E since none of the other choices made any sense (off-hand means to be ungraciously or offensively nonchalant or cool in manner) and that doesn't make sense it can't be B so it has to be A -I originally had the meaning of worldly correct as well which means to be sophisticated... -Looking up with definition got me this one wrong

An artist's preliminary sketches are often a ____of a subject; on the basis of these sketches the artist makes a decision on his or her approach to the final painting. A. reconnaissance B. caricature C. vignette D. pastiche E. cameo

*I missed this* -First I came up with the word "caricature" and saw that it was an option but then I came up with the word "provisional" meaning that the work was still in progress and incomplete and that made more sense because "caricature" implies that something is a bad representation of something or a "cartoon" and there is no clue for that -I didn't know the reconnaissance also had the meaning of "preliminary surveying or research"

The classical music critic suffers the double indignity of opining on a field that the layman mostly ____________: not only must he deal with the fact that the public tends to look ____________ on critics but he must also risk coming across as ____________ a subject in which few deign to show any interest. Blank (i) A) spurns B) embraces C) misinterprets Blank (ii) D) askance E) unduly F) smugly Blank (iii) G) deeply moved by H) dispassionate about I) condescending towards

*I missed this* -I interpreted everything right but missed the blank I did not expect to miss -I filled the last blank first because it is the only one that worked -I picked "unduly" because I thought "askance" couldn't work; it means to look at disapproval or suspicion; but unduly means "excessively or improperly" and now that I see that realize that it can't work -The first blank I was stuck between A and C but realized that there is nothing which gives the clue of being misinterpreting but the last sentence says "deign" which means someone thinks they are above someone else and so A works best

*Strategy: You must break down idiomatic phrases like "not usually so", which means "usually he is the opposite of whatever is being implied".* To Simon, not usually so (i)____, their bantering talk seemed (ii)____. Actually their exchanges masked underlying (iii)____. Blank (i) A. inscrutable B. unperceptive C. perspicacious Blank (ii) D. amicable E. exasperating F. hostile Blank (iii) G. antagonism H. assumptions I. geniality

*I missed this* -I picked D and G which are correct but picked the wrong answer for the first blank -A is wrong because inscrutable refers to ideas that are hard to understand. The clue is "not usually so" meaning that he is "usually" able to understand motives and feeling. So the right answer is B because he is usually not so "imperceptive". C is the opposite of what we are looking for.

*Key: When I change my mind on the direction of a sentence, I need to make sure to plug in everything and see if it still makes sense.* During a decade of (i)____, social scientists sought to (ii)____ the idea of the family as a healthy and stabilizing force, and replace it with the view that the family was (iii)____. Blank (i) A. moderation B. conservatism C. iconoclasm Blank (ii) D. promote E. debunk F. iconize Blank (iii) G. moribund H. progressive I. paramount

*I missed this* -I shouldn't have missed this -The second blank is easier to come up with because of the clue "replace" so the word that best goes with that is "debunk" -The if they are debunking then the first black would be C -The last blank should be easy now they are debunking or trying to say this claim is not true anymore and this ideas doesn't exist and so G works best

Since the Romans failed to subjugate the tribes in Northern Britain, they built a wall in a (i)____ attempt to (ii)____ the natives. After all, what wall can (iii)____ the determined? Blank (i) A. seemingly overoptimistic B. thoroughgoing C. successful Blank (ii) D. intimidate E. exclude F. barricade Blank (iii) G. deter H. conquer I. circumscribe

*I missed this* -So I picked A and G which make sense -For the second blank originally I had picked D but that makes no sense since there is no explicit clue for that -I picked F but that is wrong because "you barricade yourself" meaning you "made a barrier for yourself" but that doesn't make sense here so "exclude" is the right answer *Here is the perfect place to see if a word is used in a specified way or not. Do be an idiot, this is something that should come naturally and you should rely on it. Barricade usually happens when you are trying to keep something out like zombies or something. We know the answer is not D so the answer must be E. We can't find a way to eliminate it but we have eliminated D and F (since the Romans wouldn't try to "barricade" the tribes, that's an action 1) the tribes would have to perform and 2) they wouldn't do that because they weren't intimidated to begin with. So... even though I may not like the choice that is left, I have to go with it.*

The game of chess is an example of a ___ information system: the pieces sit inertly on the board until the players move them according to known rules. A. interactive B. passive C. cybernetic D. disruptive E. logistic

*I missed this* -The clue is "inert" and the best answer for that is "passive"

It cannot be denied that without creative reasoning it would not have been possible to (i)____ of classical physics. Yet classical physics has no contribution to make to the understanding of (ii)_____. This kind of (iii)____ is surprisingly common in logic as well as in life. Blank (i) A. dispute the value B. lay the foundations C. understand the basics Blank (ii) D. creative reasoning E. other sciences F. the arts Blank (iii) G. circular reasoning H. inflexibility I. symmetry

*I missed this* -The first blank was easy to come up with because of my own "clue" of "create" "come up with" -The second blank I knew because of the clue "yet" had to be opposite A and the choice that worked best was "D" -The last blank was tough; I knew it couldn't be circular reasoning because that would imply arriving back to the starting point; I thought it couldn't be "symmetry" because two things being juxtaposed would have to be the same -I was also unsure of "inflexibility" which really in GRE has the meaning of "unwilling to change one's mind" so symmetry makes sense because there is some sort of a parallel. Pick it and move on *Define each choice carefully based on GRE vocab when stuck between two choices.*

A highly intelligent person often thinks (i)____; a few snippets of information can trigger a (ii)____ conclusion that might not stand up to closer, and (iii)____, scrutiny. Blank (i) A. too deeply B. too warily C. too quickly Blank (ii) D. firm E. labored F. hasty Blank (iii) G. slower H. precipitous I. overt

*I missed this* -The first two blanks are intuitive to figure our -The last blank has to be the opposite of the first two "quickly" and "hasty" so the answer is "slower"

The preliminary review concludes that Dr. Stuart needs to (i)____ his argument with more experimental data. In its current avatar his thesis is so (ii)____ that it must be deemed (iii)____. Blank (i) A. define B. bolster C. culminate Blank (ii) D. exigent E. monumental F. slight Blank (iii) G. succinct H. profound I. inadequate

-The first and last blanks were easy to figure out but the second blank I had to think about because I didn't know the word "exigent" -So I left it blank and came back to it with a fresh mind and came up with my own word "sparse" and picked slight

*Key GRE interpretation: "At once" means two opposites are true at the same time. For ex: at once friendly and aloof.* At once pioneering and ____________, her work owes a substantial debt to her predecessors, many of whom grant she has done ____________ to advance the field. Blank (i) A) subtle B) visionary C) derivative Blank (ii) D) little E) much F) nothing

*I missed this* Level: Very Hard -For this first blank I went with "visionary" because of the "and" and the word "pioneering" usually these two things would imply a synonym but the word is "derivative" off the idiomatic phrase "at once" which implies opposite -But these guys "grant" (agree to acknowledge) that her work has advanced the field (little and nothing are too close in meaning to be the right answer)

Perhaps there is nothing more to the album than its case that experimentalism into uncharted sonic landscapes did not ____________ with Stockhausen. Or perhaps its forays--many of which could rightly be dubbed sophomoric--into the avant-garde also lead to the ____________: that to create an unprecedented sound one has to ____________ a discernible melody. Blank (i) A) come full circle B) culminate C) die Blank (ii) D) unsettling conclusion E) unwarranted hypothesis F) uncharacteristic rebuttal Blank (iii) G) choose to create H) forgo producing I) subtly embed

*I missed this* Level: Very Hard -The last blank is easiest to fill and the first can be arrived by realizing that Stockhausen is a person and that this experimentalism did not "end" with him -The second blank is hard; I focused on explicit clues and went with "unprecedented" and picked "uncharacteristic" (distinctive and not typical) but it's apparently wrong because there is no clue for "rebuttal". E doesn't work so D is the best option

*When there are no clues to support a word and it is suspect you have to start thinking if the word you aren't considering could be the answer.* *Strategy: On hard TCs don't just go with choices which seem right because they would be in real life situations. You have to think outside the box and try to see the least obvious choices and put them in context of the given sentence. Going for the least shitty option is different than just picking the choice which seems obvious and "normal" but has not GRE logic or clues backing it.* *Watch for idiomatic phrases which may signify a figurative meaning.* James Clerk Maxwell once remarked that the best scientists are, in a sense, the ____________ ones; not hemmed in by the ____________ of their respective fields, they are able to approach problems with a(n) ____________ mind, so to speak. Blank (i) A) adaptable B) revolutionary C) ignorant Blank (ii) D) myopia E) preconceptions F) inertia Blank (iii) G) fertile H) rational I) empty

*I missed this* Level: Very Hard -I approached this by figuring out the second blank which is the easiest "preconceptions" (my own word was bias) -For the last blank the word I came up with was "fresh mind, clear, original" and the best choice would've been "empty" but I didn't follow that and picked "rational" instead because it looked like a good choice -The first blank I completely ignored "ignorant" because it made no sense -The key is the last phrase "so to speak" so that whatever we put in the blank has to be closest to "not knowing anything" and being "open-minded" but the choices we pick are not to be taken literally

To view a film by Torneau is to enter the auteur's mind. That his reality fails to correspond in salient ways to that of a "normal" person does not ____________ —even if Torneau is incapable of escaping his own head. To appreciate his work, the audience simply has to indulge the director his ____________and leave at the theater door its own ____________. Blank (i) A) justify censorship B) serve as a rebuke C) preclude a connection Blank (ii) D) solipsism E) spontaneity F) chauvinism Blank (iii) G) demands for coherence H) expectations of resolution I) presumptions of the world

*I missed this* Level: Very Hard -I did the second blank first and go the answer "solipsism" -Then I approached the second blank and crossed of H and was between G and I and pocked G because it looked like a good answer choice but I can see why I would make sense since everything is taking place in the film-makers head so you have to let go of what you are used to or know about the world; there isn't a clue that says that people would "demand" for coherence; nothing says that the film-maker is "incoherent" -The first blank I went with least shittiest answer choice which was C and is that is true then G can't make sense because you can "make a connection"

*For hard ones learn to eliminate based on which are "too severe" or need to be used in certain cases. Define, define, define on a piece of paper and see if a word applies to a certain situation better.* Very few veteran critics tend to be ____________ the recent decade in cinema. Nonetheless, based on movie reviews many could easily come to the conclusion that the last ten years were indeed banner ones. Once the province of lettered intellectuals, a few even household names (Pauline Kael comes to mind), the role of the movie critic has been ____________ by those lacking any notable credentials. With this flood of veritable tyros opining from the rafters, a movie's overall rating—as compiled and tabulated by popular Internet sites-often times confers a(n) ____________ on a film, an assessment that posterity will most likely deem specious. Blank (i) A) enamored of B) condemnatory to C) unsympathetic Blank (ii) D) duly appropriated E) amply filled F) irredeemably disgraced Blank (iii) G) aura of nostalgia H) mantle of inviolability I) patina of respectability

*I missed this* Level: Very Hard -I figured out the meaning this one pretty easily but missed the blanks -The first blank is easy to figure out -The second is tough. D can be eliminated because "duly" means "rightly" which doesn't make sense since the person here disagrees with recent tyros. So you are left between E and F and both could work but F is *too severe* Amply has to be with "ample" which means "more than necessary or more than needed. -The last blank is also tough because I was stuck between H and I. Inviolable means that something is sacred and cannot be violated. The last one just says "respectability". So I works better than H *which is too strong and needs to be used in a special circumstance.* Also patina means a "gloss" or "outward" appearance of something. And the person would agree with that since he/she believes that recent ratings are not true

Hollywood has a way of ____________ reality, thereby completely altering not only past events but our perception of these events. To take but one example, after watching Milos Forman's Oscar-Winning Picture, Amadeus, audiences smugly walk away "knowing" the Italian composer Antonio Salieri sordidly did away with the divine genius Mozart. That such a story is at best a ____________ is, to them, not even a(n) ____________. Blank (i) A) subtly hewing to B) casually distorting C) fundamentally transmuting Blank (ii) D) sensation E) contrivance F) canard Blank (iii) G) fabricated deception H) implausible matter I) remote possibility

*I missed this* Level: Very Hard -I interpreted the meaning correctly and did the last blank first and picked I -I did the second blank first and was between E and F and picked F because contrivance means to "something which is unoriginal" or "done to deliberately achieve deceitful or clever scheme" -The first blank I was between B or C and picked B because I thought that fundamentally was too strong here -But for these the clues have to come from the TC. The first sentence says "thereby completely altering..." this sounds more like "fundamental transmuting" rather than subtle change *Made the mistake of not picking based off clues*

An element of ________ on the part of the audience is interwoven into the multi-era saga, for two actors portraying the same character at different phases of life are distinguishable enough that the audience is able to discern differences for which the mere passing of years cannot account. A) surprise B) foreboding C) disbelief D) confusion E) predictability

*I missed this* Level: Very Hard -I interpreted this sentence as telling us that the audience could pick put everything and so things aren't really that much of a surprise and then based off the answer choices picked E -But the right answer is C because they can see that it would not be possible to portray different characters that look that similar and age non-realistically; A doesn't work because that would imply the audience didn't know the differences -My choice E doesn't work because that would imply that the audience itself is predictable

What tradition has long known, science must labor through its usual rigorous protocols to arrive at the very same assessment. Concerning learning in infants, recent findings (i) ______________ this trend: the timeworn yarn that babies are (ii)______________ —and oftentimes disregarding—stimuli from their surroundings has been turned on its head; although (iii) ______________ exhibiting a mastery of their respective worlds, infants are constantly conducting experiments—very much like scientists themselves—testing their limits vis-a-vis an environment at once enchanting and frustrating. Blank (i) A) buck B) uphold C) underscore Blank (ii) D) passively receiving E) subtly parsing F) actively misinterpreting Blank (iii) G) far from H) known for I) potentially

*I missed this* Level: Very Hard -The last two blanks are easy to fill the first one is tough -When you get to a situation like this you really have to start thinking and using logic about interpreting the sentence -Basically tradition knows this trend, but science still have to make progress getting to it through its experimentations. If that is the case then science is lagging behind and so must be "bucking" resisting this trend *Also notice that B and C are too similar this should send off alarm bells in your head*

*Key: The word "TELLING" on the GRE has the same function as the words "SURPRISING" and "INTERESTING". What comes in the blank is different than what is expected. For example "Isn't it interesting or surprising that..."* It is telling that a politician long adept at inhabiting any role that will serve his immediate purpose has been able to (i) ______________ a disgruntled electorate, an outcome that perhaps speaks more to the electorate's (ii) ______________ nature than it does to his ability to be (iii) ______________. Blank (i) A) easily placate B) only repel C) shrewdly court Blank (ii) D) fickle E) disaffected F) docile Blank (iii) G) persuasive H) candid I) misleading

*I missed this* Level: Very Hard -I picked the last blank first and picked "persuasive" since it was the only blank that made sense -Then I went to the second blank and was between E or F. D didn't make sense since this TC isn't about the fickle minds of the voter. Rather D is the trap answer. Eventually I picked "disaffected" *because of the clue* "disgruntled" which means "unhappy". *This is a good example of what one should do when stuck on choices and has an explicit clue which pushes you to the right choice (but one you didn't necessarily think could be right).* -The first blank is tricky but this is a GRE trick. The word *telling* functions the same way I have come to expect the word *surprising* to function. These words denote that something is the case which is opposite of what people think about. Normally we would think that suave politicians would be able to control any electorate but it is "telling" or "surprising" that they are not able to control them (that's why B is the answer). This is "telling" fact is speaks more to the nature of the electorate rather than the politician's ability to be persuasive

Whether repression has come from the church or from a totalitarian state, science has always been an imperiled endeavor, but to claim that it will only flourish in times of libertarian rule is not a(n) ____________ conclusion. A(n) ____________ government is not the same as one that actively takes an interest in funding science - and the latter may well be, in some respects, ____________. Blank (i) A) superficial B) ineluctable C) tentative Blank (ii) D) despotic E) aloof F) permissive Blank (iii) G) corrupt H) inviolate I) autocratic

*I missed this* Level: Very Hard -I shouldn't have missed this as it was the easiest one to make sense of -The word that I came up with were "for sure" for the first blank and picked ineluctable -The second blank I based off of "libertarian" and "active interest" and thought the opposite of active interest would be being aloof but the word is "permissive" allowing someone to do whatever they want and that is closer in meaning to both -The last word is based off the phrase "in some respects" meaning not be to taken literally on the whole "autocratic" makes sense here because of the words "totalitarian" from the first part of the sentence

That traditional forms of media—despite considerable variance in the quality of writing—tend to report on a range of issues (i) ____________ by the demands of the readership should (ii) ____________ those who believe that the demise of each media outlet signals a lamentable reduction in the scope of news reported. Blank (i) A) unbounded B) circumscribed C) sensationalized Blank (ii) D) discourage E) mollify F) rile up

*I missed this* Level: Very Hard -I thought this was one of the easier ones to figure out and chose B because of the clue "reduction in scope" -The second blank I thought would follow from this and F would make the most sense -I still don't understand why the second blank is E. Apparently it is because the people who lament want a limited scope in reporting

*DO NOT change blanks just because you change your interpretation. If the clues lead to you filling in a blank (especially the first blank you filled in), this is the blank which is usually picked correctly.* It is ____________ that the short story, regardless of its acclaim amongst certain members of the literati, has ____________ amongst the public. All the more so, because the novel, in some ways an inherently more demanding form, continues to be popular amongst lay readers who apparently subscribe to the trite credo that bigger is better. Blank (i) A) unsurprising B) encouraging C) telling Blank (ii) D) languished E) burgeoned F) imploded

*I missed this* Level: Very Hard -Originally I came up with "languished" for the second blank based off the interpretation that "only a few people like it" and "novel is preferred" -I had trouble with the first blank and then picked "telling" but by then I had changed my interpretation of the second blank too and picked "burgeoned" -"telling" means revealing or to reveal something not know; it would make sense to keep "languished" based off every other clue -"Telling" in this case means "interesting"; isn't it interest that although it is liked by the literati it is... In this case it is the best choice that works

The subjectivity inherent in travel is aptly captured in the range of styles used by different writers. For Hemingway, writing eighty years ago, the experience of travel—regardless of how momentous—was rendered in (i) ____________ observations, a style many of today's writers studiously (ii) ____________. Then there is travel writer Pico Iyer, for whom a simple stroll through an airport can beget sentences bursting forth with as many semicolons as revelations. Who thought the terminal could be so (iii) ____________? Surely not many writers today. Blank (i) A) prosaic B) aphoristic C) sardonic Blank (ii) D) avoid E) lampoon F) cultivate Blank (iii) G) irrevocably wrenching H) wildly unpredictable I) endlessly fascinating

*I missed this* Level: Very Hard -The second and last blank are easy to figure out -The first is tough because of A or B. I picked A thinking that is will be opposite to "Pico" who thinks everything is fascinating. -The answer is B. Aphoristic which means "short and witty". Aphorisms are short and pithy sayings. This works because the other writer is not short and pithy but bursting with "semicolons" and "revelations" and keeps going on and on and on -Prosaic means "dull and uninteresting" or "lacking imagination". Which we don't have clue for

*Key: when you first settle on a word and have a clue backing it up then there is no need to doubt yourself again. You are following the right protocol. When stuck between two words, the best outcome is one which has an explicit clue. Use that to your advantage.* For charities operating in the developing world, when noble impulses (i) ______________ into mere (ii) ______________, vapid slogans rear their heads and we witness a further deterioration in the very situation such high-mindedness had initially sought to (iii) ______________. Blank (i) A) devolve B) morph C) coalesce Blank (ii) D) quixotry E) fraud F) altruism Blank (iii) G) limit H) prevent I) ameliorate

*I missed this* Level: Very Hard -The second and last blank are easy to fill -The first is tough because either could work. So one should look for explicit clues that make it clear and the clue was "further deterioration" which means turning a situation badly so devolve works best -But I went back and changed it to "morph". Don't ask me why but I did even though I picked devolve first

Pared down over the years to the point of ____________, Stockton's prose nevertheless preserves the writer's insights - indeed they are ____________ than ever. Blank (i) A) austerity B) abstraction C) artlessness Blank (ii) D) keener E) more vague F) more formal

*I missed this* Level: Very Hard -The second blank is very easy to figure out but for the first blank I was down to two choices A or B and went with B since I thought the blank had to be exact opposite to the second blank but I forgot to follow my strategy of when narrowed down to two choices "look for explicit clues" -Here the explicit clue is "pared down" which means cut down and originally when I had read the sentence the word that I had come up with was "terseness" -Now if I had followed my strategy of comparing the word that I came up with + the explicit clue I should've picked A and not gone with B *For blanks that require exact opposites the context will be obvious. Do not forsake the strategy though, because it will lead to the right answer almost all of the time.*

*When I'm doing a first time read and my intuition without warrant comes up with a word for the blank WRITE DOWN THAT WORD. If you can come up with absolutely no other reason for what the blank should be go with the closest synonym for the word you came up with.* That we may become flaccid after our rivals have been vanquished, and we are surrounded by those friendly to our interests, is in no way a(n) ____________ observation. Still, history is rife with examples where a sense of ____________ pervades once a people has achieved victory. Yet, even were this insight more ____________, few would take notice, as human nature is wont to ignore future threats in times of prosperity. Blank (i) A) pithy B) trite C) astounding Blank (ii) D) duty E) camaraderie F) complacency Blank (iii) G) widely circulated H) clearly unassailable I) hastily dismissed

*I missed this* Level: Very Hard -This second blank was the easiest to fill so I did that -The third was the next easiest to fill so I did that H and I did not make much sense -The first blank was tough, I knew the meaning to all of the words but still picked the wrong answer ultimately because I thought pithy and astounding were decently close in meaning and I couldn't think of another word -The reality was that the first time I read the sentence that answer that came in my head was "surprising" but I forgot this as I dragged this on. The reason it that the word itself that make much sense with the sentence structure because of the last blank but it does once you see that the reason it is not astounding is because its been repeated so many times through history but the last blank throws you off

Amongst business school students exists a(n) (i) ______________ that the more (ii) ____________ the material, the less bearing it will have on their respective futures. This unspoken notion is by no means (iii) ____________ by the finding that business leaders who read voraciously cite as their favorite works books that offer practical guidance on how to succeed. Blank (i) A) apt supposition B) representative trend C) tacit belief Blank (ii) D) topical E) esoteric F) theoretical Blank (iii) G) supported H) redressed I) challenged

*I missed this* Level: Very Hard -The word I came up with for the first blank was "conception" and the best option is "tacit belief" based off the clue "unspoken notion" -The second blank has to be opposite of "practical guidance" from the last sentence and the option that goes best with that is "theoretical" -The last blank for me was tough and I couldn't make sense of the sentence anyway. I thought it couldn't be "challenged" because to be challenged would be to oppose what the first part said and the first part is in favor of being "practical"; I thought it couldn't be "supported" since that didn't make sense and flow with the sentence because the second part DOES seem to support the first part and so I went with H -- so because of this the answer is "challenged" implying that "the second part does not challenge or make void the first part, it in fact supports it." -Redress means to make something bad you did better or to correct and error. There is no clue for that in the sentence. If there is no clue for something explicitly *YOU MUST CROSS IT OFF*; *even if you don't like the choices that you are working with then, you need to pick one of them.*

Many are quick to contend—albeit falsely—that the personal essay is a(n) _______ genre: historically, the form has always played second fiddle to the more brash acts, such as the novel or long-form journalism. A) recent B) introspective C) marginalized D) understated E) moribund

*I missed this* Level: Very Hard -This one you have to plug in one at a time to see what works -The steps to follow is to cross off all the choices that can't work; A, B, and C if you know understated means "subtle" or "subdued" -Then you are between C and E. Notice that the people are contending "falsely" so they won't say that the genre is marginalized since they have to be the opposite of what comes after the comma. I did this interpretation correctly. So the answer the works best but may not be the word you came up with is "moribund". Question Type: *This is the question type of... The case is this or contends to be this but the "reality is" that "it has always been like this."*

Many so-called social playwrights are distinctly ____ ; rather than allowing the members of the audience to form their own opinions, these writers force a viewpoint on the viewer. A. conciliatory B. prolific C. iconoclastic D. didactic E. contumacious

*I missed this* -Here we need something that says something about "not allowing you to form your own opinions" -I narrowed down to D and E and picked E but that is wrong since it means "stubborn" -D is correct because apparently didactic means "intending to preach or instruct"

For an artist of such circumscribed talent, Mario was given ____________ attention, many connoisseurs ____________ over works that warranted nothing more than a(n) ____________ glance. Blank (i) A) scant B) sporadic C) scrupulous Blank (ii) D) poring E) passing F) faltering Blank (iii) G) derisive H) cursory I) tentative

*I missed this*; I spent 6 minutes on this Level: Very Hard -The last blank is easiest to figure out, H -In my mind I kept thinking of the application of the idiomatic phrase "For a _____ of such" which is usually used in the form -- "for a bad writer...he sold a lot of books", "for a bad movie... it did quite well in the box office". Ultimately I did not pay attention to my instincts -The second blank follows from the first blank and is a direct continuation because of the comma... so the answer is "poring" *Clue: "for" means a shift in GRE. What comes before the comma must be opposite in meaning to what comes after.*

Vermeer is able to imbue his paintings with a saintliness verging on the ____________, a quality that is in sharp juxtaposition to the ____________ of his subjects: a milkmaid preparing breakfast, a servant tidying up a messy kitchen. Blank (i) A) artificial B) numinous C) hagiographic Blank (ii) D) similarity E) conviviality F) banality

*I missed this; but it has an important GRE interpretation clue.* Level: Very Hard -The second blank is easy to fill since it is the only choice that makes sense -For the second blank I came up with the interpretation that even though the work is boring the painter is still "glorifying" their lives but the clue really was simple "saintliness verging on _____" the original word that I came up with was pious but that didn't make sense -The word would be deity and that makes sense -The question to ask is that the word isn't artificial but is it that: 1) the painter is idealizing the subject 2) to treating it like a God and the answer is this

Biological clocks are of such (i)____ adaptive value to living organisms, that we would expect most organisms to (ii)____ them, and, indeed, we find that such clocks are virtually (iii)____. Blank (i) A. meager B. ambivalent C. clear Blank (ii) D. eschew E. possess F. select Blank (iii) G. ubiquitous H. unknown I. compulsory

-For the first blank the word I came up with was "importance" and so C made the most sense -For the second blank the word that I came up with was "have" so E fit well -The last blank before I even looked at the choices, I picked "ubiquitous" as my word and it was there but I thought compulsory could work as well. Ultimately I did all other TCs and came back to this one and decided that: 1) the clue was "we would expect *most* organisms to have them" 2) another clue is *indeed* which is GRE for a continuation of thought, so if you expect *most* organism would have them then indeed you will find that biological clocks are "everywhere" -So the key is to realize that the third blank works in conjunction with the second and has no bearing on the first

*Perfect example from the first blank about how I need to think these TCs through on the GRE. The answer might not be the one that one would think of but makes sense based off the context and clues.* Science advances (i)____ as (ii)____ change abruptly and we are forced to stop and reorient ourselves to view old information in new ways. Blank (i) A. exponentially B. inexorably C. jerkily Blank (ii) D. paradigms E. axioms F. continuities

-For the second blank I came up with the word "views/ideas" and the best answer to that was paradigms; another clue would be "old information in new ways" so there is a clear shift in "perception" -The first blank was tough but the clue was "abruptly" and "forced to stop and reorient" and that word that went best with these was "jerkily"

Edward was understandably upset that he had lost the position, but he was (i)____ by the conviction that he had done nothing to (ii)____ the dismissal. Blank (i) A. saddened B. miffed C. consoled Blank (ii) D. merit E. mar F. delay

-For this one I debated about the first blank; clearly for the second blank nothing besides D makes sense and so based off that the first blank is C but I thought may the logic might also flow this way: "he was understandably upset that..." but "he was not understandably upset that..." but GRE logic can't be that convoluted at least not for the easy ones -Here "but" is the clue; we need something opposite to "upset"

Unwilling to admit that they had been in error, the researchers tried to ____ their case with more data obtained from dubious sources. A. ascertain B. buttress C. refute D. absolve E. dispute

-Here I narrowed down to A and B and defined ascertain to have a very specific meaning: to make certain through effective measures that something is the case and that's not the meaning that we are going for so B makes the most sense

In a fit of ____ she threw out the valuable statue simply because it had belonged to her ex-husband. A. pique B. goodwill C. contrition D. pedantry E. prudence

-Here I narrowed down to A and D and the word that I came up with was "rage" and the word closest to that is A (even thought I didn't know what it meant) because to be pedantry means to be "petty or care about the small details"

Since ancient times sculpture has been considered the prerogative of men; women sculptors have, until recently, consistently met with (i)____, or even (ii)____. Blank (i) A. discouragement B. vilification C. concern Blank (ii) D. ridicule E. approbation F. tolerance

-Here based off the construction of the sentence, the second blank has to be more intense than the first blank and that's why B can't word

After an initially warm reception by most reviewers and continued ____ by conservative thinkers, Bloom's work came under heavy fire. A. criticism B. endorsement C. denigration D. counterattack E. refutation

-Here because of the "after an initially...", I knew that this was a shifter but the key was paying attention to the blank and in what scope the blank is functioning -This is a shifter but it is contracting two things, one before the blank and one after the blank so the word that one needs to look for must be positive since what comes after the blank is negative. So, I picked endorsement -Another hint is that A, C, and D are way too close to each other to pick w/o a clearer hint

The formerly (i)____ waters of the lake have become (ii)____. So even though the waters are teeming with life, fish are no longer visible from the surface. Blank (i) A. murky B. stagnant C. pellucid Blank (ii) D. tranquil E. verdant F. turbid

-Here the clue is in the last part and the two words are opposites -"are no longer *visible*"; that would mean we can't look in the lake because it is not "lucid" or "clear" so C makes sense -The second blank has to be the opposite so "turbid' makes the most sense -rely on the clue don't try to think other things like "turbid" might mean "turbulent" and the opposite of that is "stagnant"; even if you arrive at that logic the clue is "no longer visible" which strongly applies to "pellucid" rather than "stagnant"; if something is stagnant doesn't mean that you can see through it *synonym: the clue must stand strong. so that choices you pick must have the absolute meaning and no breathing room.*

It is a common complaint that people today have a short attention span. But is it that people are (i)____ if the television camera (ii)____ a view, or is it that the (iii)____ from one angle to another has trained the viewer to expect variety? Blank (i) A. satisfied B. fascinated C. impatient Blank (ii) D. lingers over E. cuts short F. rapidly changes Blank (iii) G. constant shift H. delay in moving I. inability to move

-I did the last blank first and came up with G because of the clue "short attention span" -I knew because of "or" the second blank would have to be opposite of the last and so D makes the best sense -For the first blank I chose the clue "short attention span" which would imply being "impatient" and so I went with that

Even thought Malcolm X eventually went on to sever ties with the members of the Muslim Brotherhood, the Brotherhood believed that, far from being a heretic, Malcolm X endorsed a view of Islam that was more doctrinally ________ than that of the more ostensibly orthodox sects. A. pure B. renowned C. compromised D. vague E. differentiated

-Key is to break-down things in your own words -First -- "far from being a heretic" = "he was not a heretic" -"ostensibly orthodox" means a bit sarcastic "supposed" meaning not really orthodox -So we need a word closer to orthodox = "pure" -Trap is choice E if you don't break down the sentence

*Strategy: When there is no clue for the explicit, pick a choice that is all-encompassing.* Homo sapiens, the proud splitter of the atom, inventor of the electronic computer, (i)____ of the genetic code may be humbled by a lowly (ii)____ of the sewers and soils - the microbe. Blank (i) A. designer B. author C. decipherer Blank (ii) D. creation E. denizen F. rodent

-The first blank is easy C -The second was a little tough because the words weren't printed clearly but the choice is to pick between E or F -I went with E because it is an all-encompassing answer and there is no clue in the TC that the word "had" to be a "rodent" -Also a microbe is not a rodent

*Strategy: When you come up with a word but don't get an exact match and then are left with two options, realize firstly that one of them is completely wrong, and then look to the one that could work and there is nothing in the TC that suggests otherwise. This must be the answer.* In keeping with his own (i)____ in international diplomacy, Churchill proposed a personal meeting of heads of government, but the effort was (ii)____ , as the temper of the times was (iii)____. Blank (i) A. peccadilloes B. aversions C. predilections Blank (ii) D. doomed to failure E. instantly accepted F. considered worthwhile Blank (iii) G. amicable H. auspicious I. inimical

-The first blank was trick because the word that I came up with was "style" and thought peccadilloes might work but there is nothing to suggest that predilections couldn't work

When the (i)____ weather forced us to stay indoors, we resorted to (ii)____ board games to pass the time. Anything, however (iii)____, was better in our present troubled state of mind than sitting in silence. Blank (i) A. congenial B. restorative C. inclement Blank (ii) D. inane E. exhilarating F. challenging Blank (iii) G. time-consuming H. vacuous I. versatile

-The first two blanks are easy enough to fill -The last blank I thought maybe because of the "to pass the time" G might work but that is not the case. Here the third blank is referring two the second blank and so the answer is "vacuous" to go with inane

In their day to day decision making, many senior managers do not follow the apparently (i)____ model favored by orthodox management experts, but rather rely on intuitive processes that often appear (ii)____ and (iii)____. Blank (i) A. conscientious B. normal C. rational Blank (ii) D. thoughtful E. cerebral F. capricious Blank (iii) G. logical H. iconoclastic I. deliberate

-The last blank is easiest to fill since it must be opposite "orthodox" so H works -The second blank only makes sense with F -The first blank must be the opposite of "capricious" and so "rational" works best

Through the 19th Century, the classics of Western Civilization were considered the (i)____ of wisdom and culture, and an (ii)____ person - by definition - knew them well.

-The last blank is easy to figure out because it's simply the best choice -I didn't know the word "frontispiece" but thought at first that based off my word "pinnacle" it would be the answer *But repository wouldn't be wrong either and so at this point it's a matter of figuring out which is correct and all-encompassing. There is nothing that suggests that repository wouldn't work and so pick the safest option and move on.*

The immune system is capable of distinguishing self from other at the cellular level. After grafting, unless the immune system is effectively (i)____, there is a (ii)____ of lymphocytes in the lymph glands; the newly produced lymphocytes then move in to (iii)____ the foreign tissue. Blank (i) A. primed B. suppressed C. activated Blank (ii) D. reduction E. proliferation F. stasis Blank (iii) G. stimulate H. regenerate I. attack

-The second blank and the last blank were easy but the first I was between "primed" and "suppressed" -Ultimately the clue I used was that it had to be the opposite of the second blank which is "proliferation" and so suppressed works best

*Strategy: "all but______" means "almost"; "all but impossible" means "almost (or basically) impossible". If you come across an idiomatic phrase you are unsure of, think of examples that you know and have used." Archaeology is a poor profession; modest sums are available for excavating sites and even more (i)____ amounts for preserving the excavations. As a result many sites that are still to reveal even a fraction of their potentially vital information have been (ii)____ and left to the forces of Nature. Re-opening such sites in the future will be all but (iii)____. Blank (i) A. paltry B. controversial C. abundant Blank (ii) D. abandoned E. overworked F. denuded Blank (iii) G. trivial H. impossible I. rewarding

-This is not tough but the last blank can be depending on if you know the idiom "all but____", which means "almost". And so H makes the most sense "trivial" means "petty" and that doesn't make sense and I is wrong too

With his sub-four minute mile Bannister broke a psychological barrier, and inspired thousands of others to attempt to overcome seemingly ____ hurdles. A. insurmountable B. inane C. trivial D. traumatic E. ineffable

-This one I was stuck between A and E because I thought the clue was "psychological barrier" -But ineffable means "unable too extreme to be described in words" and that didn't connect that well and A made the most sense

While unwilling to forgo the benefits of standardized testing, many European countries are seeking to (i)____ individual (ii)____ which state examinations with their (iii)____ growth have bought in their train. Blank (i) A. increase control over B. diminish the check on C. increase awareness of Blank (ii) D. spontaneity E. limitations F. persecution Blank (iii) G. randomizing effects H. tyrannous growth I. empowering actions

-This one was hard because I couldn't make head or tail of the syntax but got it right based off the pure logic of the sentence -The first part is a shifter because of the word "while..." so we need something that is opposite in scope of "unwilling to let go" and B works best because of "diminish" -The next blank follows from the first and is a direct continuation and so the choice that works best is D (the hint for this is also, because it is the opposite of "standardization") -The last blank was tough but it is circling back to the original point of state examinations being standardized and "unwilling to let go" so H makes sense because of "tyrannous"

Major philosophical (i)____ about morality, identity and rationality, for example, can often be (ii)____ by thought experiments: short and simple expositions that pose an abstract and complex problem in a concrete manner with all the (iii)____ factors removed. Blank (i) A. certitudes B. dilemmas C. dogmas Blank (ii) D. mimicked E. illuminated F. evoked Blank (iii) G. extraneous H. inherent I. pivotal

-This was a tough one for me but I arrived at the last answers. My thinking was: -The last bit really clarifies everything. Basically its talking about simplifying something and the best choice for the last blank is "extraneous" -If something is being simplified then the second blank has to be "illuminated" -if that is the case then the first blank would have to be closer to something which is hard to understand or is open to debate and the word closest to that is "dilemmas"; also dogma and certitude are too close in meaning ; also if something is "abstract" and "complex" then A and C don't much sense either

People who seek advice from (i)____ often find that what they are told can seem true, because these seekers of information attribute significance to some predictions and ignore others. The mind seeks to make sense of predictions that, in themselves, have no (ii)____ value, and thus it becomes difficult to prove that the forecasts are (iii)____ . Blank (i) A. experts B. philosophers C. clairvoyants Blank (ii) D. special E. general F. legal Blank (iii) G. genuine H. specious I. accurate

-This was tough when I was doing it but I arrived at the right answer -The first blank is easy because of the clue "predictions" -The second blank was tough but the word that I came up with was "significant" and the word closest to that was "special" -The last one is easy because "specious" is the only word that makes sense

People from all over the world are sent by their doctors to breathe the pure, (i)____ air in this mountain region to counteract the (ii)____ effects of their urban existence. Blank (i) A. insalubrious B. soporific C. invigorating Blank (ii) D. deracinating E. stimulating F. debilitating

-We know that the two blanks must be opposites of each other -The second blank can only be F and so the first blank must be the opposite of that so C works the best -insalubrious is the opposite of what we are looking for

Traditionally (i) _________ in their criticism of the hazards of nuclear power—which are undeniably pressing—many environmental groups failed to cite any viable energy alternatives; still, these very groups often (ii) __________ the success of "green technologies," innovations that, until very recently, were able to provide only a fraction of the power required to sustain a populace. Blank (i): A) muted B) vociferous C) misguided Blank (ii): D) derided E) trumpeted F) condoned

-Yet another one where breaking down instead of reading over and over again and freaking out will yield better results... Breakdown: -Environmental groups are critical of nuclear power but have offered no viable alternatives -But (still...) these guys ______ green technologies, which are now good -This is now so easy to deal with: What word in first blank best goes with critical "vociferous" and we know the second must be a DIRECT OPPOSITE since there is a "still" which is a shifter... "trumpeted" -Notice why "misguided" doesn't work. The TC tells us that the environmentalists are right in their criticism "hazard are undeniably pressing", so these guys can't be misguided at all

The British-led force's landing at Gallipoli made for such a(n) (i) ______ foray into the Ottoman theater of World War I that Max von Oppenheim, a German political adventurer in league with the Ottomans, believed that the attack must have been a (ii) ______ and that Britain had saved her better infantry for an imminent landing upon an area that was (iii) _______. Blank (i) A) strategic B) clandestine C) unpropitious Blank (ii) D) resounding success E) mere feint F) major provocation Blank (iii) G) not so heavily exposed H) similarly situated I) unevenly fortified

Level: Very Hard -At first I thought the first blank was "clandestine" but eventually changes my mind -The second blank is the easiest to fill and I picked E -Based off that I picked C for the first blank. Getting that meaning that the attack was to unfavorable that the guy thought this must be a feint -The based off this meaning I picked G; I would be the opposite since they were just defeated in an area and so must be looking for "safety" and G makes sense

*Strategy: Perfect example of why you should consider a word that you don't know. If there is a choice that works and keep the logic of the sentence correct then you should go with that choice since you know its meaning. But if the word you know the meaning of doesn't match up with the explicit clue in the TC then you have to consider the choice that you may not know.* Unlike her predecessor, Mayor Williams would not ____________ any impertinence from her subordinates. Even a ____________ comment she tended to construe as one full of ____________. Blank (i) A) discountenance B) elicit C) brook Blank (ii) D) seemingly innocuous E) clearly tangential F) somewhat ambivalent Blank (iii) G) subterfuge H) prolixity I) contumely

Level: Very Hard -For the first blank the word that I came up with was "brook" and that was an answer choice -The second blank the word that I came up with was "innocuous" and that was also an answer choice -The last blank was the hardest because of the choices G and I. Initially I thought that G was the answer and then realized based of the clue "impertinence" that the right answer has to be closer to "rude; bold; brash" and subterfuge means "deceit used in oder to achieve an end" and there is no explicit clue for that, so I went with "contumely" instead

When researchers follow the scientific method, the absence of ____________ proof by no means suggests a theory lacks validity. Indeed, no theory is ____________ : each can always be subject to further testing and scrutiny, and thus, by definition, remains ____________. Blank (i) A) ineffable B) sufficient C) irrefutable Blank (ii) D) cherished E) porous F) unassailable Blank (iii) G) equivocal H) suspect I) provisional

Level: Very Hard -For this I was able to come up with a word for each part of the blank: "solid", "incontrovertible", and "unsolved/tentative"

While Vorotsky sometimes indulges in sentimental notions, surely his entire body of work does not warrant the label ____________. A) trite B) acerbic C) treacly D) melancholic E) tawdry

Level: Very Hard -I came up with my own words of "mawkish" and "maudlin" meaning "overly sentimental" and narrowed down to C and D -D just means "sad" but nothing implies that and the word has to be closer to "overly emotional" and the word closest to that is "treacly"

The bias for ______________ has crept into the current school of physics: superstring theory provides such an all-encompassing--yet tidy--packaging of current streams of thought--quantum physics and Einstein's theory of relativity, among them--that many scientists have been beguiled by the simplicity of the theory into blithely discounting the paucity of data. A) thoroughness B) inconsistency C) elegance D) aesthetics E) artifice

Level: Very Hard -I read the sentence but crossed out the middle details and came up with the words "tidy" or "simple" -The narrowed down to "thoroughness" and "elegance" -I eventually eliminated thoroughness because it means to do something very diligently and carefully it does not mean "all-encompassing" -Then I also saw the word aesthetics which is somewhat close to elegance and realized that it must be there to trick people and then decided because of the clue "tidy" that elegance must be the word to go with

Lambert, in his latest thesis, is guilty of (i) ____________ Nietzsche's conception of eternal recurrence, a scholarly transgression that results mainly from his propensity to (ii) ____________ multiple sources. That his interpretation seems (iii) ____________ may indeed obscure the fact that he liberally combined ideas drawn from numerous works, many of them contemporary, a fact that, in part, accounts for the dubious validity of his overall project. Blank (i) A) equivocating upon B) misconstruing C) undervaluing Blank (ii) D) enjoin E) conflate F) misquote Blank (iii) G) tentative H) cohesive I) disjointed

Level: Very Hard -I went with the second blank first because of the clue from the last sentence "combined many ideas" -I then did the first blank and did it based off "picking the least shitty option": B "equivocating" or "undervaluing" didn't make sense -The last blank was tough but the clue was "may indeed obscure" so we need a word to imply that his "cohesiveness" is obscuring the fact that a lot of this is just put together from multiple sources

The question as to what constitutes art is hardly a ____________ one. Today, artists exist whose main goal seems only to subvert work that no longer warrants the trite tag "cutting-edge." Once the proverbial envelope is pushed even further, the public inevitably scratches its collective head - or furrows the collective brow - thinking that this time the "artists" have ____________. That very same admixture of contempt and confusion, however, was not unknown in Michelangelo's day; only what was considered blasphemous, art-wise, in the 16th Century, would today be considered ____________. Blank (i) A) perennial B) contemporary C) controversial Blank (ii) D) served their purpose E) gone too far F) failed to provoke Blank (iii) G) hackneyed H) reverent I) tame

Level: Very Hard -The first blank is easy based off the clue in the last part of the sentence. The TC is basically comparing past to the present -The second blank is E since there other two don't make much sense -The last blank can be tricky because I was between G or I. I went with tame based off the clue "blasphemous". And really because this blank worked as a mini TC with the last sentence. Some might think hackneyed might work because of the first part of the sentence but the last blank is the independent of that.

*Strategy: When debating a definition write it down so you are not just repeating it nonsensically in your head over and over again.* *Strategy: When the word you came up with is not one of the answer choices don't waste time repeating and holding onto that word try to see what you are missing in the meaning that will allow you to come up with another word.* However much the economist trumpeted his ____________, his accurate prediction of a major downturn was not as ____________ as he led the public to believe; for years he had been prophesying fiscal doom. Blank (i) A) affluence B) veracity C) prescience Blank (ii) D) uncanny E) unambiguous F) provident

Level: Very Hard -The first blank is easy to figure out, the second one is tough because I was debating between D or E. It was also tough because for the second blank the word that I came up with was "severe" which wasn't an answer choice -I knew it couldn't be F since provident means being wise in regards to the future -What worked for me was writing out the definition so that I wasn't just repeating it in my head over and over again. Unambiguous means not open to interpretation, having one clear and exact meaning and that didn't really fit in -Uncanny means eerie in an unsettling way and when used in the sentence it makes sense because of the clue "he had been prophesying for *years*" so this wasn't new to people and thus wouldn't be surprising or catch people off guard

*Strategy: shifters can also works backwords, for example the first blank: we want something for the first blank that is opposite what comes after the word "but" which is the "entire edifice of western thought".* Heinrich Feyermahn, in insisting that Galileo did not fully uphold the tenets of scientific rationalism, does not ____________ the Italian astronomer, but rather the very edifice of Western thought. For if Galileo is the purported exemplar of rational thinking, and yet is ____________, then the history of science cannot be understood as an endless succession of scientists carrying out their work free of all-too-human biases. Thus, Feyermahn admonishes, in faithfully chronicling the sweep of science in the last 300 years, historiographers would be ____________ not to include the human foibles that were part of even the most ostensibly Apollonian endeavors. Blank (i) A) exclusively implicate B) partially repudiate C) fully espouse Blank (ii) D) found wanting E) considered enlightened F) dismissed as inconsequential Blank (iii) G) prudent H) remiss I) contrarian

Level: Very Hard -This is a big TC to get through and ultimately boils down to picking choices that are the least shittiest -The last blank is the easiest to fill since remiss is the only word that works -The second blank could be a little tricky but the clue is "yet" which means that we are looking for the opposite of something positive like "rational thinking" and so E couldn't work also inconsequential (quite the opposite) doesn't work since HF seems to like Galileo and believes him to be an "exemplar"; so D works best and also because of the connotation about "human foibles" implying that no one is perfect -The first blank is tricky because of the choices A and B; repudiate doesn't work since HF doesn't seem to be "denying the validity of Galileo"... neither is he doing this _____ action partially (which means part of a whole); what the blank is trying to get at is that HF is not exclusively "implicating" Galileo but (key word) the entire edifice of Western thought; so A works best

Carefully couching his words in the most diplomatic language possible, so even those (i) ______________ to his cause could hardly construe his words as a (ii) ______________ , the city councilman offered an ultimatum to the (iii) ______________ group of protesters camped outside the City Hall. Blank (i) A) indisposed B) sympathetic C) impartial Blank (ii) D) panegyric E) broadside F) prognostication Blank (iii) G) defeated H) querulous I) dishonest

Level: Very Hard -The first blank is the trickiest and initially I picked B because I thought "even those..." implies someone clues to him -The last blank was the easiest to I picked H since G or I don't make much sense -For the second blank E seemed like a good choice since people are upset and protesting and something isn't being "construed" correctly; F doesn't make sense and D is a formal praise which doesn't match up -So I finally went back to the first blank and thought about A or B and cancelled C (which means to be unbiased); I re-read the sentence to make sense of it with both the options and decided that A made the most logical sense; this guy is "covering" his words in such as a way that even those against him would not be able to interpret his words as being a "verbal attack". Once I went through this process, I realized that B could simply not work

*Strategy: The first blank is an exemplary instance of how to get hard blanks which you can't come up with a word for and which don't give words that make complete sense with the sentence structure. Eliminate based on definitions that make no sense whatsoever.* She gave him a(n) ____________ look that was not so much ____________ as it was ____________. Blank (i) A) knowing B) encouraging C) unequivocal Blank (ii) D) accusatory E) approbatory F) fastidious Blank (iii) G) egregious H) tentative I) admonitory

Level: Very Hard -The last blank is a must to figure this out and the only answer choice that makes sense is "admonitory" which means to warn to express censure -The second blank follows from the idiomatic design on the sentence which is saying that it wasn't so much as _____ as it was "admonitory" meaning that you have to look for something which is more intense than admonitory and that word that works best with that is "accusatory". I was happy relied on my intuition for this syntax -The first blank I was stuck between A or C and then decided that the look was obviously a little "equivocal" and not clear and nuanced so C couldn't work also "encouraging" didn't make sense with the other choices

A successful revolution, whereby those who have unseated the previous government are able to usher in an irenic transformation, is not (i) _____________. Nonetheless, scholars, and even the public at large, persist in believing that the American Revolution, while clearly successful, bequeathed a(n) (ii) _____________ that all restive nations need only follow to enjoy effective self-rule. One must look no further than the sweep of the 20th century, to realize the (iii) _______________ of this conception. Blank (i) A) the norm B) an aberration C) an archetype Blank (ii) D) template E) notion F) maxim Blank (iii) G) futility H) ambiguity I) folly

Level: Very Hard -The second and third blanks are easy to fill -The first blank was tough because of A or C but I went with "norm" rather than archetype which has the connotation of being a "paragon" or model that must be copied rather the TC is telling us that, what occurs during revolutions is not the "norm" quite the opposite happens

Public officials recently announced plans to (i) __________ noise pollution in parts of the downtown. Aside from citing the usual costs of enforcing ordinances, the city also referenced reports indicating that residents had become (ii) ___________ the hubbub, a finding that, for the most part, is not (iii) ___________ the dearth of media coverage on the issue—when a municipality is responsible for disgruntled citizens, the newspapers are sure to follow up with a flurry of coverage. Blank (i) A) scrap B) curb C) condone Blank (ii) D) enraged by E) flustered by F) inured to Blank (iii) G) incompatible with H) reconciled with I) mitigated by

Level: Very Hard -The second blank is easiest to fill and the last blank has the clue that "there is a dearth of reporting" which means that this must not be an important issue and so G works best based off the overall meaning -The first blank was a little tough because I was debating A or C but A can't work because it is referring to "scrapping" or doing away with the sound and we are looking for the opposite of that and so C works best

That the web may seem some ____________ phenomenon, hardly rooted in the physical world, is a notion clearly ____________ by the existence of "server farms," sprawling forests of metal that, figuratively speaking, provide the backbone of the Internet. Blank (i) A) ethereal B) ephemeral C) faddish Blank (ii) D) buttressed E) betrayed F) unsupported

Level: Very Hard -The word that I came up for the first blank was "ethereal" and it was one of the choices so that made it easy -The second blank was a bit tougher since E or F seemed like they could work but I asked myself why "unsupported" couldn't work and I couldn't come up with a good answer -On the other hand, betrayed has a specific meaning in that you are trying to cover something up and then someone catches it or it is revealed but that doesn't fit the meaning here so I went with "unsupported"

For someone so ____________ in his delivery, Quentin was remarkably relaxed during his presentation, any trace of affectation gone. A) arbitrary B) guileless C) studied D) fastidious E) conceited

Level: Very Hard -This was not bad since I approached it with my strategy of trying to come up with my own word but that didn't work so I narrowed down the answer choices -In tough TCs there are more clues than necessary and you have to check the answer choice to see which one is there a synonym or close match for? -The clues were either opposite of "relaxed" or synonym of "affected" which in this case means unnatural or put on to impress and the word that went best with that was "studied"

A new school of thought has it that innate talent can be conveniently ____________ a series of readily ____________ factors - Mozart's genius then is no divine blessing of the type conferred on a select few, but is simply the result of a patriarchal father who stressed, above else, thousands upon thousands of hours of grueling practice. Blank (i) A) reduced to B) misattributed to C) measured by Blank (ii) D) intrusive E) quantifiable F) pervasive

Level: Very Hard -The first blank is supposed to be the trick but it is an idiomatic phrase and I recognized it and went with the right answer which is A (it definitely was not C) -The second blank I was stuck between E and F but being pervasive means being in abundance everywhere and that doesn't fit the meaning especially when the explicit clue was "thousands and thousands of hours" so I went with "quantifiable"

Monarchial reigns ____________ tended to be more ____________, dynastically speaking, than those royal courts in which palace machinations had not become a quotidian affair. In the latter, a pall of complacency would fall over the kingdom so that if suddenly there were an earl with an axe to grind, so to speak, his path to usurpation would be ____________. Blank (i) A) marked by intrigue B) characterized by hubris C) weakened by attrition Blank (ii) D) imperiled E) volatile F) robust Blank (iii) G) largely unobstructed H) a treacherous one I) hardly assured

Level: Very Hard (took 8 minutes) -The key here is to write down key points are you are going through them because it is convoluted and so you don't want to go over things again and again -Basically the sentence is saying that royal courts are not used to "day to day machinations" and so monarchial reigns must be the opposite and must be inured to it. So "robust" makes sense for the second blank based off that the first blank can be filled but it is hard because one would not know the not-so-well known definition of "intrigue" to mean "deceit" so you have to eliminate. There is nothing which explicitly says anything about hubris and there is nothing which suggests that the monarchical reigns were constantly attacked so to be done by attrition so you have to settle with A. *This is a hard part of learn and inculcate for the test.* -The last blank can easily be filled. If the royal courts aren't used to machinations then they can easily be defeated. So G works best

Some minor governmental bungling in the environmental sector can ____________ what many watchdog organizations are declaiming as gross negligence: indeed entire swathes of once fertile land now gone barren ____________ flaws in state policy. Blank (i): A) certainly mitigate B) clearly misrepresent C) hardly account for Blank (ii): D) cannot solely be attributed to E) can eventually be traced to F) result directly from

*Clues matter, and I relied on them on this one.* Level: Hard -This is an example of something where your natural intuition is to blame the government but the clues say something else: -The government "bungled" or messed up in a "minor" way but the watchdogs are claiming that it was "GROSS negligence"... -A small mistake can hardly account for such a large negative result and cannot solely be attributed to just state policy... especially since the mistakes were "minor" -Notice the "colon" which continues the same thought as the first part of the sentence; so both blanks mirror each other

*This is a lesson in how to translate to make convoluted easier* Most see Gutjens's latest book as a(n) _______ his previous works - yet he has always made it a habit of confounding expectations, and so any of his novels could bear this tag. A) coda to B) elaboration of C) departure from D) synopsis of E) inducement of

*I missed this one* -This one is tricky to translate -We have a writer whose latest book is expected to be a(n) ________. But this writer always surprises or confuses the reader so this book is going to be like his last works. -In other words, after "yet" what we have is people saying that his books are going to be similar to his previous books so the blank has to be "departure from"

*Strategy: When you narrow down to two choices and they look similar, try to really question why one can or cannot work (especially for harder level questions). Look for specific clues and ask yourself "what would most people pick based off their intuition and is the GRE test makers trying to throw you in the wrong direction".* Giacomo's concerti, much like the composer himself, were a ____________ affair. Fits of passion would, without warning, give way to sudden idylls, as though the composer had been trying to ____________ his inner conflicts. Only in his later works, which are far more abstract, does he eschew trying to capture his inner states. Blank (i): A) mercurial B) rambling C) torrid Blank (ii): D) exorcise E) foreshadow F) mirror

*I missed this* Level: Hard -The first blank is easy to fill out -The second blank I narrowed to D and F. I picked exorcise which means to remove from the body based off my "own word" but the clue for this blank comes later in the TC, the author later eschewed trying to "capture" his inner states -What is closer to "capture"? exorcise or mirror and what would most people go with? Probably "exorcise". But the clue works best with mirror -Also think about how exorcise is used even figuratively. You would use it to denote an action of "pulling out and expelling/getting rid" of something. Is the composer doing that?

*Strategy: Anything but = "Not"* Many claim that the 19th Century institution of snake's oil—the peddling of items that seldom live up to their vaunted claims—is anything but moribund; one need only look so far as the Internet to see that the tradition is ____________. A) thriving B) transient C) peripheral D) diminishing E) counterfeit

*I missed this* -This is not really tough although I fell for the "anything but" trap... *Anything but = NOT* -This makes the TC obvious now

Cryptozoology is predicated on a notion that is every bit as ___________ as the very quarry it aims to study: one cannot disprove the existence of that which does not exist. A) mysterious B) irrefutable C) cautious D) elusive E) fundamental

*I missed this* Level: hard -I don't think I came up with my own word first! -There is not context clue for mysterious (having a quality or importance not apparent to the senses) -The breakdown is that this study aims to prove the existence of something that does not exist or disprove the existence of something that doesn't exist. I had narrowed it down to A and D but it seems elusive rather than mysterious which is too broad anyway elusive means you are trying to find something but it escapes your grasp! *Here I should've used my breakdown of the options left and see which makes the most sense*

The travelogue is a thorny genre, even for seasoned writers, for one must ______________ a curious balance between inspired navel-gazing and reportage with a cosmopolitan slant. A) boast B) deconstruct C) effect D) inhibit E) forsake

Level: Hard -Here I narrowed down to A and C; A means to brag but there are no clues which suggest that the writer has to brag. Only that he/she needs to "strike" and the one that comes closest to this is "effect"

*Strategy: Careful with one blank hard TC, which part multiple parts to the sentence and might be used to throw people off. Cross out what is unnecessary.* Inspiration rarely leads to great writing unless coupled with a(n) ________ regimen, one which affords writers ample opportunities to experience a flash of insight. A) tedious B) exacting C) unexpected D) inconsistent E) widespread

*I missed this* Level: Hard -My initial thought was that it can't be "tedious" because that means the task is boring and and lacking in interest to cause weariness; I thought it couldn't be "demanding" since an exacting schedule would not give much time for "flash of insight" since you would have to be focused; D, and E made no sense... I thought it would be C since something unexpected would match with "flash of insight" which is also unexpected but GRE would probably put that as an option to trick most of the test takers... -The key here is to notice "regimen", which is a prescribed course of action laid out. Exacting would make the most sense. And the sentence already says that "inspiration" alone cannot lead to good writing so something else will. The second part of the sentence is meant to throw people off! -Also think about it this way: -This is a shift question because of unless, so what comes in the blank must be opposite to "inspiration"; the best choice is "exacting". *This technique may be more useful when dealing with one blanks with very little clues. Identify the type of question type; if it is a shift then try to look for the opposite that makes sense in the sentence.*

*Strategy: When stuck on a TC with no clue, answer is based on the best clues that you think are available. Do not try to overthink and go outside the sentence. The clues have to be in the sentence for you to answer it. Go through each choice one by one and look for EXPLICITLY mentioned clues.* This question is similar to the one about "Alfred P. Knopf. You don't know what to pick so look for THE explicit clue and match it. In Speak, Memory, Vladimir Nabokov's autobiography, the author seems as ____________ an ornate prose style as with communicating, often in excruciating detail, the ____________ of a singular life, describing his family's fall from aristocracy following the Bolshevik Revolution. Blank (i): A) taken with B) inimical to C) unaccustomed to Blank (ii): D) apotheosis E) vicissitudes F) generalities

*I missed this* Level: Hard -I couldn't crack the first blank at all even though the second one was easy. I ended up paying too much attention on relating it to the second blank but in this case the clue for the first blank was "excruciating detail" -I overthought this... A breakdown that could've led me to the answer for the first blank would be: This guys is talking in "excruciating detail" so his prose must be "detailed" and "florid and ornate" that means this guy liked an "ornate style" hence "taken with" works best -When stuck on a blank like this. The strategy should be to go through each choice one by one and find an explicit word or group of words that lend support to it. Do not try to overthink by bringing in outside knowledge. In this case the explicit clue was "excruciating detail"; I thought that the person might be "unaccustomed to" since people do not get used to changes in their circumstances for the worse but this is NOT mentioned anywhere in the sentence; "inimical" means hostile or a hindrance to and you might think a change in circumstance would be "perceived" as being hostile but that is also not mentioned anywhere. The only effing clue written down is "excruciating detail", don't be an arse and overthink and not go with the logic that you've been practicing. This is tough for you and so it will be everyone, your job is not to lose your shite and bungle this up.

*Strategy: Do not be misled by multiple work answers. Always see if the word you come up with makes sense and you can find a clue for it. If not then resort to looking for an alternate clue.* Whereas the incumbent's opponents feverishly worked around the clock, digging up seemingly irrelevant information only to contort a(n) (i) _________ incident so that it appeared unequivocally damning, the incumbent himself resorted to no such (ii) _________ and preferred instead to calumniate his opponents during highly publicized debates. Blank (i): A) benign B) unambiguous C) disgraceful Blank (ii) D) desperate subterfuge E) concealed outpouring F) subtle promotion

*I missed this* Level: Hard -Ok so this should not have been too hard -My first choice was D but I went with F based off what I thought was the clue but there is a trap here. *When you have multiple word choices, pay extra attention* -I thought that the opponent wanted to do "highly publicized" debates and so didn't want to stoop to the opposite of that. And this may have worked if a better choice were not available and F says "promotion" the incumbent is not promoting he is playing by "trickery" -D works not only because it gets to the meaning of what the incumbent is doing which is "deceit" but also because there is a clue for it "working feverishly around the clock" and "irrelevant information" shows desperateness. Notice also that the second blank refers to the actions of the incumbent not the opponent!

In regards to the polarity of the nation's political biases, the media peddles the same tired ____________ , hoping that the repetition of a conventional idea will lead people to passively accept it. A) auguries B) epiphanies C) tropes D) anodynes E) deceptions

*I missed this* Level: Hard -So I narrowed it down to C and D -Anodyne means something which is inoffensive; not likely to provoke dissent (Magoosh says that the GRE definition is something which relieves pain) -Trope is a figurative play on words -Could be either but I missed the details of how to use anodyne it can be used an an "adjective" ex: anodyne music, meaning inoffensive music. But here we are looking for a noun "anodynes" would then mean a pain relieving medicine

Our (i) ______ the visual world—from the most startling colors to the tiniest distinction between two shades of grey—is only possible through the finely tuned (ii) _______ of eye and brain. For without light striking the retina, there would be no image for the occipital lobe to project, in a manner of speaking, onto the part of mind responsible for our visual awareness. And without the occipital lobe to (iii) ________ the cacophony of visual input being relayed from the eyes, there would be no coherent picture of reality to perceive. Blank (i) A) apprehension of B) habituation to C) repudiation of Blank (ii) D) collaboration E) perception F) differentiation Blank (iii) G) suppress H) reform I) manage

*I missed this* Level: Very Hard -The first blank was tough to figure out but I settled with "habituation to" since I thought it would mean "acclimation to" also I didn't know that "apprehension" could mean to perceive. The word of my own that I thought about but struggled to come up with was "our understanding of/being able to perceive". If thought of that was habituate doesn't make much sense. The clue for this is *from startling colors to the tiniest distinction*; this really implies the sense of "perception" being able to make out the differences. If I were stuck between A and B I would've used this but I was dead set that apprehension only had one meaning. -The second one is easy to figure out because there are multiple things going on but if you *group them together* they are essentially working with one another so collaboration makes sense -The last one was tough for me but I knew it couldn't be "suppressed" and "reform" really has a different connotation of "reforming behavior, etc" so I looked at the details and the clue was that the OL is responsible for "projecting" and the word that works close enough to that is "manage"

Amongst Irish-American playwrights of the early 20th Century, her work stood out as a(n) ____________, not so much because of its striking originality but because other contemporaneous works tended to be ____________ on most social issues. Her plays, by contrast, allowed the audience to come to its own conclusions, a technique that foreshadowed much of 20th century theatre. Blank (i): A) exemplar B) precursor C) anomaly Blank (ii): D) unyielding E) dogmatic F) reticent

*I missed this.* Level: Hard -First I did the second blank -Then I picked the first blank to be "anomaly"; because it suggests that her work was "different" from other writers -Then I stopped to keep thinking and thought "foreshadowed" was a clue to the first blank and picked precursor which was wrong. The first blank is not talking about being a foreshadow it's focusing on the fact that writer is "atypical" or "different" from the others *I'm waiting too long and thinking too much. I should just go with the word that I come up with first. This would allow me to avoid missing too much by being messed with the clues.*

*Strategy*: When stuck between two choices: 1) Ask yourself if this sentence is a shifter 2) If so what is the main argument that you need to find the opposite of? BEWARE of parenthetical phrases especially on hard questions. See what most people would go for and why the opposite answer may be correct. The answer will boil down to GRE logic of black & white choices (exact opposites)* Because the corporation has suffered consecutive quarter losses, at times arbitrarily terminating jobs, employees fondly recall the ____________ days. A) prodigal B) predictable C) insufferable D) archaic E) halcyon

*I missed this.* Level: Hard -I narrowed down to B and E and picked "predictable" because I thought the clue was "arbitrarily terminating jobs"; but that is just a parenthetical phrase. The main argument is "Because the corporation had *suffered* losses". -Since this is a shifter we need an exact opposite! The opposite of suffering from the options we have is "halcyon"

*Strategy: Once I come up with a word before/after that is a synonym for the one given. I have to go with it.* In August 27th, 1883, the Indonesian island of Krakatoa, home to a highly volatile volcano, disappeared overnight in a display of stunning geological prowess that continued ____________ even after the island had vanished, as a series of massive seismic shocks created a tsunami with waves of 150-feet high that traveled nearly a thousand miles. A) furtively B) haphazardly C) undiminished D) hypothetically E) retroactively

*I missed this.* Level: Hard -I read this the first time around and narrowed to C and E -I pondered over it for a long time and the word that I came up with was "unabated" and the word that closely resembles that is C not E -Retroactively means "done after the fact" - adverb - after the fact; think of "retrospective" thinking about things once they are done

*Strategy: make sure to see that your answer choice makes sense as a whole. It is one sentence, so the whole has to make sense. Trap choices are usually easier choice words that everyone will know.* The effects of radiation are ______________: only after many years, once a chronic disease surfaces, do people realize they had, at some point in their lives, been exposed. A) debatable B) insidious C) obvious D) indiscernible E) benign

*I missed this.* Level: Hard -I thought insidious was the trap answer and the main clue is that the disease is unable to be made on until later BUT the key is to note the continuation ":" -If the disease is indiscernible then you won't be able to see or find out about it at all; but here we do find out about it once it surfaces, so "insidious" works better. Also note that the effects are harmful since they lead to a disease. Also note that there is an opposite to the word that is the right answer "benign" so the right answer has to be "insidious". It is way too well hidden. -Also, sometimes it's better not to overthink and go with gut feeling. Here it's talking about a disease and that is best used in conjunction with "insidious". The "trap" answer choices are usually the easier sounding words like "indiscernible" rather than "insidious" ------ *I even came up with my own word "insidious" and that was part of the answer option. When I do this, I have never been wrong. Only go through the "hard process" when: 1) You can't find the word you are looking for 2) You can't get the exact choices to work and have to debate between 2-3*

*Strategy*: When doubtful on the hardest GRE TCs, be especially vigilant on looking for the RIGHT clue. Even if you get the meaning question yourself this way: 1) Is this a shifter? If so what am I looking to find the opposite of? 2) Who is the main subject what is the action and what is that action guided by. Why is the person taking that action? Careful about extra information that follows (or is in parenthetical phrases). 3) Pick from your narrowed down list. The choice may not be obvious to you. Ask what the test maker is thinking.* For all her brilliance, Ada was undone by a(n) _________ that led her to disparage those who took any speculation too seriously, averring that the philosophical mind worked best when it did not attach itself too strongly to any one line of thought. A) arrogance B) insouciance C) misconception D) extravagance E) agnosticism

*I missed this.* Level: Hard -My thinking was that the main clue was that she did not "attach herself to any one line of thought"; She was undone by something which -> led to disparaging so the clue is not "disparaging" the clue would be something that led her to "disparage", so I had the clue right -I thought agnosticism worked best because it means having a "doubtful or noncommittal attitude towards something"; I clearly didn't know this word that well... There is nothing in the sentence denoting that she is "doubtful" -So I had narrowed down to A B or E and A doesn't work. If I knew "agnostic" more carefully I'd maybe avoided it... -Here the clue is very tough... She disparaged people for taking speculations "too seriously"; So we need the opposite of that. The opposite of serious is carefree and B is the answer.

*Strategy: If you are 50/50 between two words but can break then down and come up with a reasoning as to why one may be the answer over the other. Go with the one that your logic suggests is correct. Do not just pick on randomly.* The Olympic Cycling Team took their ____________ at the base of the mountain, hoping that the extra calories would sustain them during the tortuous ascent. A) vows B) repast C) umbrage D) dander E) leave

*I missed this.* Level: Hard -The clues aren't hard but the words "dander" and "repast" weren't familiar to me -My thought process was to actually go with repast first because it gave me the connotation of "breaking your fast" and also through the root "past"ure which is what cows eat, but I didn't go with my reasoning and went with the other choice. *Write this on the scrap paper when going through is process smoothly and calmly.* It'll help you see the clear connection.

Some note that the increase in the Native American powwow--an intertribal affair of song, dance, and storytelling, all intrinsic aspects of Native American culture--serves to (i) ______________ the very culture it presumably aims to (ii) ______________. They argue an overarching cultural narrative emerges, one that (iii)______________ the narrative of any one tribe. Blank (i): A) erode B) distill C) empower Blank (ii): D) foster E) undermine F) question Blank (iii): G) subsumes H) elaborates upon I) overcomes

*Level: Hard* -Here I did the last blank first since it was easier to fill and the clue was "overarching", so we need a word that is close to that and subsumes works -The other two blanks had to opposites of each other and I felt that based off of the last blank the first had to be a negative since it takes away "individuality" and the other blank had to be the opposite of that and "foster" works pretty well -Also notice that the second blank had 2 similar answer in "undermine" and "question" which were both wrong -Overall breakdown is that these powwows must be a way to demonstrate individuality and uniqueness but that is undermined by an overarching narrative which emerges -An easier method would've been to do the second blank first since it follows directly from the third and look for its opposite!

It was not out of mere ______________ that Mozart, by then well established, sought out a young Beethoven—in the latter's early work Mozart divined a genius that he hoped to, in some way, cultivate. A) contrariness B) solicitude C) surliness D) prejudice E) enlightenment

*Took me a long time but I applied my strategy of breaking down in terms of character and motives to the easiest form of the sentence possible, and it worked.* Level: Hard -I narrowed down to A, B, and E -My initial choice had been solicitude but I wanted to make sure it was correct -The breakdown the lit the bulb for me was: 1) We've got a dude (Mozart) who is really good 2) He's found someone who he thinks has great talent and feels he can help hone that talent 3) So he's not just being the "nice guy", he wants to mentor this guys because he thinks the guy has great talent -The trap answer would be "contrariness", since Mozart's already great and is seeking someone else out -Enlightenment means to realize something you've haven't before; education which results in understanding -Contrariness means being stubborn and going against other people (being contrarian); that couldn't work (I didn't know this when doing the question) -Another way to go about this TC would be to go one by one and realize that you need a word that really makes sense and is accurate and maintains the logic of the sentence. Solicitude works the best

Thomas Chippendale, the 17th Century English designer, was renowned for his ____________ style: even typically prosaic furniture pieces were ornately embellished. A) plebeian B) cosmopolitan C) rococo D) intuitive E) austere

- Here is an example of one where I didn't know the definition to the word that was the right answer but I went with it by eliminating all the rest -I thought maybe cosmopolitan could work, but stuck to the strategy of going for exact definitions -We need a synonym for "embellished" -- Do not forget to know exactly what you are looking for

In the last century, the US has truly become a ______________ nation: immigrants from over a hundred countries have created a veritable tapestry of languages. A) cosmopolitan B) dignified C) universal D) polyglot E) cherished

-Again a simple TC but one that highlights an important point -One might be fooled into picking choice "A" because a place with citizens from all over the world BUT... -Notice that the blank is defined by what follows that colon and that is what we are looking for that rest is just ornate stuff Plan: The right answer then is "polyglot"

Though she had spent years toiling away, she had again, much to her ____________, been passed over for a lucrative post that went to someone far less senior. A) delight B) disenfranchisement C) diligence D) prescience E) chagrin

-Another example of where it's not the word that I came up with but makes the most sense -My initial reaction was to try to see which of the other words mean "disappointed" and rethink my logic; I need to get better at ignoring this and picking the answer and just moving on (still, I made progress on time and spent only 43 seconds)

In spite of the (i)__________ of his presentation, many people were (ii)__________ with the speaker's concepts and ideas. Blank (i): A) long-windedness B) formulation C) forcefulness Blank(ii): D) enthralled E) bored F) gratified

-First notice that this is a shifting question and so what goes in the two blanks must be opposite. But the text completion is tricky because anything could go into the two blanks. Thus, this is the type of question that if you stare at it too long, will lead you to the wrong choice and waste time -First intuitions should be written down... Interpretation: In spite of the liveliness of the presentation, the people were bored/confused with the speaker's concepts Plan: -One could eliminate choices that do not have polar opposites: "formulation" doesn't have a polar opposite; one could notice that in the second blank "enthralled" and "gratified" are too alike in meaning and so probably are the wrong answers -Another thing would be to match original own words with that is available. The thing is that what you have down might not match exactly or that your definition is not what you know the word to possibly mean. So "bored" would still be true for the second blank. The polar opposite of "bored" in the first blank would be "forcefulness" (even though you do not know the full definition of forceful which means with vigor in this case)

With characteristic ____________, H.L. Mencken skewered the sacred cows of his time, criticizing social trends and government institutions with equal asperity. 1) hauteur 2) playfulness 3) vitriol 4) civility 5) dash

-Here I couldn't make sense of the sentence and then used my strategy to figure out the subject and what that subject was doing -The key really was to narrow in on the terms "skewered" and "criticizing" and "asperity"

As municipal water experts very well know, successive years of greater than average rainfall can lead to a dramatic surge in water usage, with the populace ______________ the very habits that had once sustained the city. A) engaging in B) returning to C) altering D) mocking E) relying on

-Here is an example of how to break down a question in very simple terms: -Rain goes us -> water usage goes up -This causes people to ______________ the habits that sustained them through the drought years... -The answer should be pretty simple: "C"

Because the defendant expressed very little ____________ for his heinous crime, the judge meted out a(n) ____________ sentence. Blank (i): A) contempt B) contrition C) apathy Blank (ii): D) charitable E) severe F) peculiar

-This is not a hard one but has a good lesson about paying attention to detail -We know this flows one way because the defendant: 1) expressed very little 2) he has done a heinous crime -So he showed little "sympathy" and so the judge would have to give him a "harsh" sentence

True, to the classically trained ear, Haydn's early works can often seem ____________, a mishmash of motifs from which anything fresh has been wrung dry by subsequent composers—to the ears of Haydn's contemporaries, however, Haydn's music was ____________. Blank (i): A) complex B) predictable C) hackneyed Blank (ii): D) refreshingly novel E) prematurely antiquated F) highly derivative

-Here is an example of something you should try to break down and make sense of by connecting with logic. Do not try to add new knowledge than what is given to misconstrue though Breakdown: Classically trained ears: find Haydn's music _____ because it's *not fresh* (why? because all the composers that came after him used all his fresh stuff... so they are used to everything). Notice that I don't even need my logic in the paragraph, just the fact that his music is not perceived to be fresh is enough and simple, which is what you should be aiming for The people during Haydn's time though ______ So the second blank will be the opposite of "not fresh" since this a shift in perception and time

Mike Tyson, during his ____________, was the most feared fighter in the world; his fall from eminence was as astonishing as it was ____________. Blank (i): A) tenure B) debasement C) heyday Blank (ii): D) seemly E) unsustainable F) precipitous

-Here is yet another example where you might try to look for a clue to the blank and will come up with your own word but that is not an option -Go with the second best choice then and what makes sense. Define each of the options and eliminate based on what can't work. Do not doubt your thinking and try to make the answers work

That the nightmarish depictions common to most early 20th century dystopian novels are exaggerated should by no means diminish the _________ power of these works, for many of the visions they conjure up are reflected, albeit in less vivid form, in many totalitarian governments today. A) synoptic B) ephemeral C) comprehensive D) apolitical E) prophetic

-Here it's getting a bit closer to harder level TCs -The key is to realize the contrast between "20th century" and "today". So essentially the works were good "predictors" for what was to come. This is a word different from what one may think of immediately but it follows the correct logic -Although prophetic wasn't my intuitive guess, the logic suggests it is the right answer and even though I didn't know synoptic it was wrong anyway. Furthermore, I didn't pick comprehensive, which just means that it "incorporates" everything. But clearly we are dealing with time and being "prescient" not "comprehensive"

Unlike the performances of her youth, in which she seamlessly inhabited a role, the performances of her later years were ____________, as though she were calling out to audiences, "look how convincingly I can portray my character." A) decrepit B) comical C) volatile D) mechanical E) contrived

-Here paying attention to the details is going to get you to the best answer -I narrowed down to D and E (A happens usually to be a trap answer as per Gruber and in this case it is, decrepit means old and since this is a shifter comparing young and old some might fall into this trap) -Mechanical means lacking imagination and done as if rote but there is no connotation in the TC that says this, thus although I didn't fully understand what contrived meant here, it is the right answer

The travel writer's ______________ towards others he met on his cross-country trip most likely endeared him only to those readers with a misanthropic bent. A) diffidence B) humility C) cynicism D) garrulity E) obsequiousness

-I got the right answer but it took some effort because reading it the first time w/ the blank I didn't know what the TC was talking about -Then I focused on the details: he is a writer of traveling and is being liked by people who are misanthropic so his attitude towards others on this trip must be "cynical"

For some time now, __________ has been presumed not to exist: the cynical conviction that everybody has an angle is considered wisdom. A) rationality B) flexibility C) diffidence D) disinterestedness E) insincerity

-Note the words "for some time" and "now" which indicates a "time shifting" question so whatever goes in the blank must be opposite to what is "elaborated" on after the colon Plan: -The second part says that the "cynical" (distrusting and disparaging) conviction that everybody "has an angle", which really simplified means that everybody has an opinion or bias is considered wisdom -So that opposite of "bias" or "angles" or "opinion" must be "disinterestedness" (a very popular GRE word) which means "unbiased"

Philosophy, unlike most other subjects, does not try to extend out knowledge by discovering new information about the world. Instead it tries to deepen our understanding through (i)__________ what is already closest to us -- the experiences, thoughts, concepts, and activities that make up our lives but that ordinarily escape our notice precisely because they are so familiar. Philosophy begins by finding (ii)__________ the things that are (iii)__________. Blank (i): A) attainment of B) rumination on C) detachment from Blank (ii): D) essentially irrelevant E) utterly mysterious F) thoroughly commonplace Blank (iii): G) most prosaic H) somewhat hackneyed I) refreshingly novel

-Notice that the second sentence is a shifter so what comes in the first blank must be opposite of what the first sentence is saying -Note also the "--" which signifies elaboration -The second and third blank can be tricky since they are in the same sentence and very close to each other so special attention must be paid attention to -Overall the text completion is saying that philosophy is more about "thinking" about things and experiences that are "ordinary" and commonplace and finding the "meaning" in these things -Also, when the answer choices has many words and modifiers, those modifiers matter a lot Plan: -The first blank should be easy "B" makes the most sense" -The second blank should also be easy given that you can pick up subtle hints that "D" is negative and "F" is not close to "finding meaning or interesting" normal things so "E" makes the most sense -What kind of things are we looking to be mysterious? Commonplace and everyday things... "G" makes the most sense "prosaic" makes "commonplace" (in writing "lacking poetic beauty"). "H" talks of hackneyed which refers to ideas and art that is "unoriginal" and "dull" and "insipid" "I" is the opposite of what we are looking for

The professor repelled many students with his ____________ asides, often droning on about some trivial academic point. A) subtle B) alluring C) pedantic D) contemptuous E) edifying F) Previous

-This is not bad but could turn into a hard GRE question if details are not paid attention to. If an answer choice has been "discursive" this would have been very tough. But you have to ask yourself, what kind of "digressions" because there is so much to the TC that is left and we see "trivial", which would imply being petty and paying too close a attention to unnecessary detail

It is (i)__________ that so many portrait paintings hang in art museum, since the subject matter seems to dictate a status closer to pictures in the family photograph album than to high art. But perhaps it is the artistic skill with which the portraits are painted that (ii)__________ their presence in art museums. Blank (i): A) surprising B) understandable C) irrelevant Blank (ii): D) challenges E) justifies F) changes

-Notice the word "since" in the first sentence (can play a role as a "shifter" or "reasoning for what preceded it"). In this case it is both - Note the word "but" at the beginning of the second sentence indicating that what comes in the second blank must be an opposite to the first blank Plan: -Attack the second blank first because the interpretation is obvious that it is the artists skill which "justifies" their presence -If that is the case and we are "justifying" the presence of these portraits in the second blank, it must be the case that the first blanks states something opposite to that effect. "understandable" can't work since it goes in the same direction as the second; "surprising" works because the second blank attempts to reconcile this surprise

In claiming that there are overarching commonalities among every culture, Jungenfreud perhaps (i) ____________ his case: while there are most likely universal belief systems and recurrent myths spanning both time and civilizations, depending on the level of (ii) ____________ of a scholar's criteria, surely not all societies display the exact same characteristics--it is one thing to say that every people has an elaborate view of the afterlife (as Jungenfreud does); it is another to show uncanny parallels between the particulars of this afterlife (as Jungenfreud fails to do). Blank (i): A) overstates B) misrepresents C) elucidates Blank (ii): D) sophistication E) specificity F) similarities

-The second blank can be a bit tricky is it "sophistication" or "specificity" -The point is to examine the details and see what works best -The clues we get are "particulars" and "uncanny" (which usually means looking at details) so here E makes more sense -Look at this question vs. the Allen Poe one, they are different in that here we did have to pick between to similar choices but they were nuanced... With the Poe one they were too similar in that context and the answer related to figuring out what "myth" referred to

In Beyond Good & Evil, The Genealogy of Morals, and many of his later works, Friedrich Nietzsche, the late 19th Century philosopher, avoided a ______________ style and instead liberally employed the use of aphorisms and pithy paragraphs -- a style whose origins can be traced to ancient texts such as the Bible and the Tao Te Ching. A) terse B) controversial C) contemporary D) unknown E) proper

-This is a really tricky TC and refers to another one above about figuring out what really is defining the blank and ignoring the other stuff (sometimes this seems counterintuitive but is the right way to approach these questions) -Notice that the first few lines can be ignored as they are just facts, figures, and numbers Plan: -Notice that the philosopher avoided a ______________ style and instead relied on short pithy statements -One might now fall into the trap of looking for an answer which means this BUT the answer we need for the blank needs to be the opposite of this BUT! notice that is not an answer choice because the blank is actually being defined by something else... "a style whose origins can be traced to ancient texts such as the Bible and the Tao Te Ching -So we now realize that "aphorisms" and "pithy" are characteristics of what is defining the blank, they DO NOT define the blank itself -The answer then must be the opposite of "ancient texts" (we can ignore the Bible and Tao Te Ching since they are facts, figures, and numbers) -The answer choice opposite "ancient" is "C"

To the senior manager, unsolicited opinions, even if the views expressed did not necessarily (i) ______________ his own views, were (ii)______________ ; thus, employees had learned to be (iii)______________ lest they no longer found themselves in his good graces. Blank (i): A) gel with B) countermand C) clash with Blank (ii): D) overt E) nettlesome F) welcome Blank (iii): G) reticent H) sycophantic I) elusive

-This is a tough TC -DO NOT sit there and try to figure everything out on the first read! 1) Read the entire thing first and get a sense of what is going on 2) Then look for which blank to tackle 3) Then come up with your own words -I missed the last blank... This TC is talking about a manager who does not like "unsolicited opinions" so the employees would have to learn to NOT give unsolicited opinions... The word that best goes with that is "reticent", which means unwilling to share one's thoughts readily

For a writer who has made a career of indulging in (i)______________, his latest work is so (ii)______________ that, despite its numerous merits, many of his most ardent readers may feel themselves pining to go back and reread the author's earlier books. Blank (i): A) clever hyperbole B) comical digressions C) outright speculation Blank(ii): D) Overbearing E) focused F) steeped in conjecture

-This is a tricky TC and is one of those where you have to plug in choices to make it work -It also highlights another main points about GRE TC -- when looking for an "opposite" you need the exact "opposite" and this choice might not make the most sense but it works with the GRE text structure Plan: -Notice that the first blank is a positive (or is something that the readers like) and the second must be an opposite -Beginning with the second blank you notice that any of them could work and so you need to look at the choices for the first blank and then see which choices form opposites -Choices "C" and "F" say the same thing so they can't work -Choices "A" and "D" are not exactly opposites so leave them -Let's say you pick "E" then check the options for the first blank and notice that the only one that makes sense it "B" -focused vs. digressions

Whereas for most of the West art is merely something to stand and gawk at, for the Navajo Indians art has a far more ____________ function; sand paintings are used to heal those who are sick—either in body or mind—and only these afflicted individuals are allowed to see the art: as soon as they recover the painting is ____________. Blank (i): A) curative B) religious C) peripheral Blank (ii): D) refurbished E) destroyed F) exhibited

-This is an easy TC but I still missed it! -An important lesson is that just because you come up with a word on your own, do no get attached to it. For ex: second blank I thought of "put away", but this isn't one of the art choices and I "made exhibit work for me" -A good technique when stuck would be to question, where have I encountered a "pure" version of this word before "to exhibit something would be to show it", which is the opposite of what we need and so even though "destroyed" sounds strong, it is the right answer

What is currently (i) ______________ civil engineers is not so much a predicted increase in annual precipitation as the likelihood that many storms will come in (ii) ______________, thereby making flooding in lower lying riparian regions (iii) ______________. Blank (i): A) worrying B) comforting C) unimportant to Blank(ii): D) more predictable patterns E) tighter succession F) greater isolation Blank(iii): G) far more likely H) somewhat infrequent I) all but impossible

-This is one of those tricky one's where you have to plug in to get right but a few things to keep in mind: -Always try to decipher the intention of the TC by yourself; in these types of questions your intuition is usually on point -When stuck look for key details that lead you in the right direction Plan: -You may notice the clue "predicted increase in annual precipitation", this means that the TC is about more flooding not less... So, for Blank (i) "B" and "C" are wrong -For the last blank then the right choice has to be "G" -Then for blank two "E" would make the most sense

Spending five minutes in the sun will most likely have a(n) ____________ effect on one's skin; spending fifteen minutes, on the other hand, can lead to burning and, over time, fine wrinkles. A) harmful B) demonstrable C) enduring D) indeterminate E) benign

-This is relatively simple but good in terms of teaching an important lesson -Notice that the word that we need refers to a list of things. When this occurs, group the many things together and then decide what the overall words describes that group is. In this case "burning and wrinkles" would be "harmful" and we need the opposite of that so "benign" works well

Advocates of anti-smoking campaigns are (i) _____________ the fact that the number of smokers per capita has (ii) _____________ since the 1970s, perhaps aware that, lest this trend reverse itself, their message, which is predicated on dire predictions, is likely to be more (iii) __________ than one rooted in reality. Blank (i): A) quick to cite B) reluctant to acknowledge C) indifferent to Blank (ii): D) remain unchanged E) been dramatically ballooning F) been steadily decreasing Blank (iii): G) harmful H) disingenuous I) efficacious

-This is tough at first but can be easy to dissect if approached with the GRE mentality -We have a group, whose predictions are based on "dire things"... So they would be "reluctant" to cite good news B & F. The reason is that if this trend were to reverse itself... then what they say will appear more "effective" than something rooted in reality (which is that smoking has gone down)

No other contemporary poet's work has such a well-earned reputation for (i)__________, and there are few whose moral vision is so imperiously unsparing. Of late, however, the almost belligerent demands of his severe and densely forbidding poetry have taken an improbable turn. This new collection is the poet's fourth book in six years -- an ample output even for poets of sunny disposition, let alone for one of such (ii)__________ over the previous 50 years. Yet for all his newfound (iii)__________ his poetry is as thorny as ever. Blank (i): A) patent accessibility B) intrinsic frivolity C) near impenetrability Blank (ii): D) penitential austerity E) intractable prolixity F) impetuous prodigality Blank (iii): G) taciturnity H) volubility I) pellucidity

-This is tough because there is so much going on so rephrase this in your own words and tackle one blank at a time -Overall the text completion is talking about a author whose writing is deemed to be very "unsparing" and "sharp" and "biting" -Notice that "of late" implies a time shift so whatever comes after must demand a different answer than the first blank -A new thing has happened "of late" the author has published a prolific amount of material, which is surprising for someone whose prose is so caustic and unsparing -Notice another shifter "yet" which implies another shift probably back in line with the first blank Plan: -First blank should be easy to solve given that one know's their vocabulary. "C" makes the most sense. -Second one is also pretty straightforward now. "D" makes the most sense the other two are garbage choices -The last one can be tricky since it is another shifter and talks about "something knew" that has happened. The only new thing that has happened is that the author has become prolific so that one that makes the most sense is "H".

Edgar Allen Poe biographers tend to fall into two camps: those who try to rescue the man himself from a macabre world in which fate had decreed nothing less than a(n) (i) ______________ outcome, and those who (ii) ______________ that very myth, treating the subject as one for whom a life of tragedy was (iii) ______________ . Blank (i): A) dire B) unforeseen C) auspicious Blank (ii): D) dispute E) hold fast to F) squelch Blank (iii): G) all but inevitable H) clearly unexpected I) hardly justified

-This is tough for the second blank -The first and third are easy to figure out especially if you read through the TC -The second can be hard... You would assume since there are two camps one would want to have "disputes" BUT notice that the first camp tries to "save the man from what they THINK is a hopeless world", and the others try to _________ this myth and say that "Poe's life was meant to be a tragedy" -What is the myth here? The myth is referring to "the world is hopeless" thus E makes sense -Also notice that dispute and squelch are alike in meaning... The answer can't possibly be two choices, so this should've made me examine carefully what to do -I think the key is to pay "ATTENTION" to the detail (like the lesson" and know what is being referred to

Crusty and egotistical, Alfred A. Knopf, the publisher, wore (i)__________ shirts from the most exclusive tailors; was a connoisseur of music, food, and wine; nurtured a garden of exotic plants; and enjoyed rare cigars. His self-assured, (ii)__________ manner, together with his insistence on the best of everything, shaped his ostentatious publishing house's image as a (iii)__________ of works of enduring value. Blank(i): A) expensive B) immaculate C) flamboyant Blank(ii): D) decorous E) irascible F) congenial Blank(iii): G) connoisseur H) purveyor I) solicitor

-This is tricky because this text completion is long and because you have to look for clues very are separated and located all over the place and not in the logical place you might think to look -Notice that the first blank could be anything, so the clue might come from the other blanks or clues from the passage itself Plan: -Note that the second blank is probably the easiest to fill because it is talking about AAK's manners, which are pointed to at the top "crusty and egotistical" egotistical matches with self-assured so the word we need must mean "crusty" and "E" makes sense -The last blank is also relatively easy "H" is correct since a purveyor is a seller or goods or spreader or ideas (solicitor doesn't work since it means to advertise) -So we notice that the other blanks are not clues in themselves so the clue must be somewhere in the text completion itself. The strategy should be to go through the text and see if any specific word gives association to the choices. We notice that the word "ostentatious" appears towards the end. This word means "flashy or showy", and the only choice that means the closest to this in meaning is "C - flamboyant"

Human nature and long distances have made exceeding the speed limit a (i)__________ in the state, so the legislature surprised no one when, acceding to public demand, they (ii)__________ increased penalties for speeding. Blank (i): A) controversial habit B) cherished tradition C) disquieting ritual Blank (ii): D) endorsed E) considered F) rejected

-This is tricky question in the sense that your opinions lead you to the wrong answer choice: speeding up is viewed as negative so you would assume that the government would want to endorse penalties. But... the right answer is different and depends on the logic of the question Plan: -First notice that this is not a shifting question -The first part of the sentence says that human nature has made exceeding the speed limit a "regularity" so A or B could work -The second part of the question says that "acceding to public demand", meaning the legislators "gave in to the public demand" to what? Well if human nature has made speeding a regularity then giving in to the popular demand the legislators would "reject" the penalties" -If this is the case then B makes more sense for the first blank since "controversial habit" would imply the opposite that this is a negative habit and needs to be stopped but the opposite happened when legislator "acceded"

Perkin's wit, surprisingly ____________ by the prudishness of his time, may not have been nearly as ____________ had he lived in an era not so prone to ____________. Blank (i): A) tempered B) overwhelmed C) untrammeled Blank (ii): D) comical E) restrained F) racy Blank (iii): G) blushing H) vacillation I) expression

-This is very hard but because it is the type of question that you have an intuition to go for a certain answer -This is yet another TC where I've missed the clue "surprising", which means to indicate that whatever is taking place must go against intuitive judgement! -Perkin's wit was "SURPRISINGLY" _____ given the prudishness of his time. Well if the time's were prudish means they didn't take talking about sexual matter lightly and so if his wit is "surprising" then it must be "untrammeled" -> "racy" -> "blushing"

Unlike classical music, which has many perennial favorites, pop music produces few ____________. A) mannerisms B) evergreens C) detractors D) affidavits E) ephemera

-This one I got the right answer but it proved to be a bit tough because of the vocabulary -One thing is to notice it is a double shift sentence, so the word that comes in the blank has to be a synonym for "perennial" -I was stuck between B and D but eventually chose B. The key is to conjure out whatever you possibly can to decipher the meaning on the word you are stuck at, and I had a good feeling about "evergreen" but also could have used logic "evergreen" mean always green -On the test go with the choice for which you can make the most substantiation (roots, feelings, whatever) -Complicated words usually have one meaning... It's the words that are commonplace that might have multiple meanings and the GRE loves to test that ("affidavit" is something that has a specific meaning and is not used often, evergreen could be mistaken for many things)

The grammar teacher ____________ at the slightest fault, sometimes going so far as to berate students for forgetting to cross a 't'. A) started B) caviled C) blanched D) beamed E) paused

-This one teaches two important things: 1) Details matter! There is a teacher and he _____ at the "slightest" fault, berating students for "forgetting to cross a 't'". The forgetting to cross a 't' is not for show it is important. Means that this teacher is petty and overly fussy/fastidious 2) I was stuck between blanched and caviled but went with the right choice B by examining the detail "vile"

J. Robert Oppenheimer's role in the Manhattan project as the architect of the atomic bomb left such a(n) (i)_____________ on the public conscience that the remaining fifteen years Oppenheimer spent (ii) ____________ nuclear weapons seem a mere historical footnote. Blank (i): A) indelible mark B) fleeting impression C) significant diversion Blank (ii): D) campaigning for E) further testing F) railing against

-This one was another closer to GRE hard level since it had so little to go one but really forced you to look at the details -A person is the main builder (architect) of the atomic bomb, so it left a ______ on the public opinion (well if he is the main guy, the let's say the response was "positive") -The next 15 years, shift in time, he spent _______ were ignored (has to be opposite of the first blank)

Parson Weems, George Washington's preeminent biographer during the president's life, is responsible for spreading many of the ____________ we today accept as the unvarnished truth. A) canards B) assurances C) disenchantments D) enmities E) certainties

-This seemed a tough one and probably closer to the level of hard GRE questions because it forces you to look at the details -Key is to note that this is a "shift in perception" as there is a difference in time Breakdown: -There is a biographer who spread _____ information -This information is *today* accepted as the *truth* (I didn't know what unvarnished was so best to ignore it) -Well... if we today accept it as being true then we must have accepted them as being false in the past -What work means close to lie or misleading fact or fabrication

That, through no fault of his own, George Cuvier, the father of extinction theory, has mostly slipped into obscurity, his name typically surfacing only in paleontology journals, is an outcome that—given his field and his claim that his work would endure—smacks of ___________. A) unexpectedness B) arrogance C) magnanimity D) irony E) insignificance

-This took me time to figure out but it is again about translating the sentence to make it easier... This entails crossing out a lot of verbiage -Breakdown: -This dude has become obscure (fade into nothingness) and the fact that his name only pops up now and then ______ the fact that he thought his work but endure

Cave paintings recently found hundreds of feet below the surface in southern France suggest that prehistoric man viewed animals as central to both his rituals and existence. Whether this focus results from the idolization or subjugation remains ____________ - for every painting of a beast riddled with spears, there exists another in which man is depicted in a far more ____________ role, arms outspread as though in obeisance. Blank (i): A) incontrovertible B) equivocal C) inconsequential Blank (ii): D) diminutive E) deferential F) sacerdotal

-This was a bit tricky at first, but the clues are a bit obvious -The second blank can be tough if you are stuck between E and F, the thing is to go with what you know and I knew deferential would mean "idolizing" so I picked that -Sacerdotal means relating to priesthood which is not the same thing as "idolizing" -Another clue is "obeisance", which would mean "deferential respect"

Legal (i)__________ initiated by the government necessitate that manufacturers use (ii)__________ in choosing food additives. Blank(i): A) entanglements B) restraints C) proclivities Blank(ii): D) knowledge E) moderation F) caution

-This was a tricky one because although the first blank is easy to figure out there are two choices that could be possible for (ii) but only one makes sense Plan: -So it's obvious that the first blank is "B" since entanglement suggests "being caught in something inescapable" or a "compromising situation" -For the second blank the word one comes up with could be "restrain" but "E" or "F" could work. But the devil is in the details... -Moderation is usually used when telling someone not to indulge in something they are using and something that can be measured. Here the manufacturers are "choosing food additives", which are whole items, not something they are indulging in. So, the answer must be "F" caution

Because reading on the Web entails quickly scanning and sorting through a deluge of information, many wonder if our level of engagement with the text has been ____________ or if the ability to read closely and carefully is one that can be ____________ if we simply spend more time immersed in a book. Blank (i): A) irreparably compromised B) tentatively disrupted C) permanently restored Blank (ii): D) fully reactivated E) further degraded F) summarily disregarded

-Took around 1:30 to figure this out but learned a good lesson from it -Here "or" in this case is working as a shifter and if that is the case then what comes after must be a "complete opposite"... That made it much easier to settle with D. This is also the reason that "tentatively disrupted" cannot work, since we want exact opposites! "fully" and "tentatively" are not exact opposites here. "irreparably" vs "fully" *This can easily turn into a very hard TC if there were answer choices that were the exact opposite. So details would matter... The word "deluge" would be negative, you would be getting overwhelmed with information! So that would be the clue to get you to the right answer.

*Strategy

-When stuck on a TC and can't figure out which blank to fill, ask yourself the following: 1) Am I supposed to look at the nuances and figure out which of the choices works best? OR 2) Have I interpreted the TC incorrectly or missed a key detail which actually refers to something in a specific way and I don't know what that thing whether it is a "myth" or "transmission" is referring to? 3) Also be very wary of you see two choices that are similar... If you have missed a key point then the chances are that neither one of those is the answer. You have to go back and look for what you missed and see if the option you didn't pick makes sense then. Usually this type of TC is less obvious than simply figuring out which of the two fits best. That's why these types of questions are probably the hardest.

Vascoux, in not exclusively ______________ the tenets of modern jazz, imbues his trumpet playing with a pathos reminiscent of the Romantic composers. A) catering to B) undermining C) debunking D) hewing to E) validating

Level: Hard -Here I narrowed down to A and D. It seems the hard questions really make you choose between two really likely choices but there is one that works the best. There is nothing in the sentence which suggests that the player was "catering" (satisfy the demands of something) to someone or something; it talks more about a style of playing which he is either adhering to or not and D makes more sense

While caffeine is well-known as a stimulant, few are aware that an excess of caffeine can actually have a ____________ effect. A) restorative B) paradoxical C) soporific D) revitalizing E) detrimental

Level: Hard -I first figured out that this is a shift and the main clue is "stimulant" so the answer has to be the opposite of that "soporific" -The trap answer is "detrimental"

For an obscure poet to have penned such a refined, poignant sonnet is not at all ____________. The sonnet, after all, has been a favored form for hundreds of years amongst the amateur and lionized alike. I would be ____________, on the other hand, had not one, out of the sheer number produced during this time, surpassed Shakespeare on a bad day. Blank (i): A) rare B) puzzling C) conceivable Blank (ii): D) confounded E) vindicated F) hard-pressed

Level: Hard -The first blank could be A or B (the context says B) -The second blank is D and so since the two blanks are related on the GRE we want exact synonyms or exact opposites; in this case synonym and so B works best

In his critique of the student's ____________ essay, the writing instructor mostly focused on ____________ details, leading many in the class to believe he was either oblivious to the subtleties of the piece or simply envious of the student. Blank (i): A) meandering B) polemical C) probing Blank (ii): D) trifling E) nuanced F) significant

Level: Hard -Here I knew the I had to tackle the second blank first and then get the opposite of it in the first -Trifling made the most sense since "nuanced" would imply that the teacher did pay attention to the subtleties which he did not and significant makes no sense -The first blank has to be the opposite of the second and mean something positive "polemical" is a sharp written/verbal attack and there is no context for that and meandering is negative and we want the essay to be good so probing makes the most sense

While some maintain that the recent proliferation of uncredited web sources will have a(n) (i) ______________ effect on scholarship, others argue that the effects will be far more (ii) ______________, claiming that academics are sensible enough not to (iii) ______________ unattributed sources. Blank (i): A) inestimable B) pernicious C) minute Blank (ii): D) harmful E) benign F) subtle Blank (iii): G) place credence in H) take issue with I) express skepticism towards

Level Hard: -Here you have to solve the last blank in order to the get the first two -The second blank you may be vacillating between E or F but there is nothing that says that effects will be "subtle" which has a different connotation that "harmless" which works because the choice in the first blank has to be the exact opposite of blank (ii) and since B is the right answer E must be the right answer

Far from eschewing a(n) ---- lifestyle and adopting one commensurate with that found in developing nations, the international humanitarian outreach has accorded its members perquisites befitting a decadent monarchy. A) noble B) austere C) conventional D) extravagant E) cloistered

Level: Hard -"Far from eschewing" in this case means They are not staying away from a life of ________? In this case extravagance; "Not avoiding a ______ lifestyle" -Example of an idiomatic phrase -Noble is a trap answer

Revolutions do little to cure a nation's ____________ - the very agitation that allowed for a change in previous rule often precludes the formation of a viable government. A) jingoism B) paranoia C) restiveness D) euphoria E) cupidity

Level: Hard -A or C; the clue is "agitation" restiveness makes the most sense since there is nothing which suggests "extreme patriotism"

Due to recent advances in technology, archaeologists have the potential of ____________, even from the tiniest fragments, the provenance of pottery shards. A) interpreting B) interring C) determining D) reconstructing E) relating

Level: Hard -Another one looking for a specific choice rather than a general answer. The clue is "provenance" of potter shards. Well if they are trying to figure out the time or age of something then they are "determining" not "interpreting" which would have a connotation of trying to figure out something which is convoluted

Fenton's motives were clearly ____________ , yet Fenton tried, in the most ingratiating way, to ____________ his innocence. Blank (i): A) aboveboard B) base C) overt Blank (ii): D) maintain E) dismiss F) hide

Level: Hard -Here "yet" is a shifter but it has a unique way of shifting; yet here refers to the actions base vs. innocent -Tackle the second blank first, he is trying really hard "ingratiating" to maintain his innocence -The first blank must mean that his motives were then clearly "bad" -His motives were "bad" BUT he tried to "maintain" his innocence

*Strategy: When between two words, you know one that fits well as opposed to the other; go with the one you know. Since there can only be one right answer then the other must be wrong.* While the aviators had hoped for no ____________ meteorological events, the weather became increasingly ____________ , with wind tossing their plane as they crossed the Pacific. Blank (i): A) crucial B) untoward C) propitious Blank (ii): D) torrid E) inclement F) predictable

Level: Hard -Here I didn't know torrid but I knew inclement and knew that it meant bad weather, so it made sense to pick it and move on

*Strategy: "could or could not" options get cancelled out* There are few ____________ thrills to be gleaned from Kafka's writing, for his characters, which typically embody ideas, are not fleshed out enough for the reader to become fully immersed in their plights. A) novel B) vicarious C) tangential D) precarious E) substantive

Level: Hard -I narrowed down to B and E -The clues talked about the characters "not being fleshed out enough" and not being able to be "*immersed* in their plights", this gave me the sense that the reader could not relate and immerse him/herself in the works and B gave a more specific reason than the broad "substantive" which really means to "substantiate" or be "grounded in reality". I also thought, just because the characters are not fleshed out doesn't mean the story isn't substantial (it could be or could not be, hence E was cancelled out) -*Also think about what most people would have picked! Most likely "substantive" and that is the wrong answer.*

History has recast the 15th century Florentine monk Girolamo Savonarola as a rabble-rousing zealot lording over the "bonfire of the vanities"; yet this so-called _______ --mainly because he directed his censure at the church--was a crusader for austerity and thus a check on a papacy that had run a course of profligacy. A) ascetic B) nonpartisan C) heretic D) martyr E) libertine

Level: Hard -I narrowed down to C and E, and figured that the clue is "mainly because he directed his censure at the church" -I went with heretic because I knew what it meant and fit well -Libertine's main definition is noun - a dissolute person; usually a man who is morally unrestrained (especially in sexual matters)

The politician spoke in a tedious, pompous manner, ____________ the entire duration of his speech. A) ranting B) anointing C) bloviating D) misconstruing E) augmenting

Level: Hard -Narrow down to A and C, and I knew the clue was "pompous manner" and the word that I came up with "pontificating" -Ranting has a connotation of talking about something that you are unhappy/angry with but this guy is just full of himself he/she isn't disparaging anybody and so bloviating (which means talk in ornate and inflated manner) is the answer

Many philosophers are known for a single utterance, an (i) ____________ saying that long outlives them. There is often (ii) ____________ in this phenomenon. While most undergraduate philosophy students can quote the 18th Century philosopher David Hume as saying "Reason is the slave of the passions," David Hume himself actually consigned this apothegm to the marginalia of his text. In all likelihood, he had (iii) ____________ he had ever written any such thing. Blank (i): A) unfocused B) epigrammatic C) obscure Blank (ii): D) subtle wisdom E) great irony F) wide renown Blank (iii): G) soon forgotten H) readily denied I) inadvertently conceded

Level: Hard -Not too bad once the last blank is solved -For the first blank even if you can't figure out which one is best, you can go through each choice one by one and find "explicit" clues in the sentence. "apothegm" clearly means a short and pithy saying so "epigrammatic" which means a witty remark fits best

The biographer who provides a ____________ of detail, even when those details are accurate, ____________ of distorting reality; the greater the number of facts that have to be fashioned into a ____________ narrative, the greater the chance that the narrative, regardless of how consistent, will in any number of ways fail to accord with what really happened. Blank (i): A) wealth B) paucity C) smattering Blank (ii): D) has an unlikely chance E) runs a heightened risk F) concocts a plan Blank (iii): G) cumbersome H) cohesive I) profound

Level: Hard -Not too hard to figure out but the key is really to look for clues that lead to the blanks -The clue for the first blank comes as "the greater the number of facts..." so we need something which implies a "lot of facts" (also notice that paucity and smattering are way too similar so either one can't be the answer) -The second blank is easy to figure out based on the clues at the end of the passage - "fail to accord" -The last blank has only one answer that really makes sense: "cohesive" based off the clue "regardless of how consistent"

With numerous exciting public works projects in the offing, residents are understandably (i) ____________ ; yet because such prodigious undertakings are inevitably plagued with numerous setbacks, much of the fervor is likely to be (ii) ____________ a heavy dose of reality. Blank (i): A) vexed B) concerned C) agog Blank (ii): D) tempered with E) intensified by F) precluded by

Level: Hard -Not too tough to crack but for the first blank I didn't know the word "agog" but I knew since this is a shift because of "yet" it had to be opposite in connotation at least to the second blank -The word I came up with was "excited" and vexed and concerned did not work in the context, so I went with the word I did not know -For the second blank "precluded" is not only too intense but it means to "stop" from being able to do, but that is not what we are going for here... We are talking about excitement which will "wane" because the projects will have setbacks

Despite the formality of the occasion, he danced with ____________, flailing his arms in the air uninhibitedly. A) decorum B) piquancy C) vitriol D) unease E) abandon

Level: Hard -So I missed the clue here and only got it right because none of the other options seemed even remotely correct -This could be a "very hard" question depending on if there were trickier choices -I noticed that the word I'm looking for is not the opposite of "formal" since I didn't find a synonym -The clue comes at the end "uninhibitedly"; means not holding back and E makes the most sense

Many high-tech CEOs, perceived as having absolute power over their respective domains, are accorded the same awe and esteem once reserved for ____________ . A) mavericks B) pugilists C) doyennes D) consiglieri E) potentates

Level: Hard -The clue here is "absolute power" -I didn't know any of the other words but potentate clearly has the same meaning

The choreographer was mainly concerned with ____________ details, as the period production required the leads to don a number of different outfits. A) frivolous B) sartorial C) practical D) sporadic E) inchoate

Level: Hard -The clue is "outfits" so this person must be concerned with what people are wearing -The answer has to be sartorial -POE also results in words that make no sense based off the clue

For someone so unjustifiably ______ success, the recently installed CEO perhaps surprised very few when his series of impractical business solutions did not ______ the floundering firm. Blank (i): A) assured of B) intrigued by C) unfamiliar with Blank (ii): D) pan out for E) end disastrously for F) reflect negatively on

Level: Hard -The clue to the first blank is "unjustifiable", so the word that goes with that has to be "assured"... He thinks he is "so good" that things will work out but this is "unjustifiable" -The second blank makes sense because the solutions were "impractical" and the firm was "floundering" so D makes sense

What is the greatest novel of all-time? Many top-100 lists have been proffered, purporting to resolve this very issue. Yet the ____________ those compiling these rankings suggests that any definitive list is not ____________. Blank (i): A) consensus amongst B) divergence of opinion of C) collective repute Blank (ii) D) far off E) forthcoming F) laudable

Level: Hard -The first blank is easy to figure out but the second doesn't have any relevant clues -E makes the most sense because it means ("upcoming immediately, or readily available", or ease in talking to others, or willing to divulge information); laudable means deserving praise, and far off suggests the opposite of what we need -Overall though, it makes sense; people can't agree on a definitive list, so such list is not going to be available in the near future since they haven't even agreed on the list! -If you guessed "imminent" for the second blank then obviously this would've made the sentence easy -Another trick might've been to see *for what isn't the clue available*

In censuring the academic committee for apparently being ____________ in appointing a chancellor, the university president mistook ____________ for procrastination--the committee had been guilty of nothing more than a scrupulous vetting of all candidates. Blank (i): A) biased B) dilatory C) overzealous Blank (ii): D) prevarication E) deliberation F) vacillation

Level: Hard -The first time around I didn't get the meaning of what the sentence was saying and so used my strategy of focusing on the subject and what the motivations of that subject are (president being pissed because he has misinterpreted diligence with procrastination)

Recent meteorological conditions in areas of the northeastern part of the country have been so ____________as to leave scientists ____________. Even those models scientists developed to ____________ these extreme outliers have been found wanting. Blank (i): A) predictable B) aberrant C) taxing Blank (ii): D) indifferent E) dumbfounded F) crestfallen Blank (iii): G) accommodate H) circumscribe I) discount

Level: Hard -The hardest blank may be the last blank and highlights why it's so important to come up with your own word. I ignored the wrong answer "discount" because the word I came up with was "account for" and accommodate worked best. -Discount means to not pay attention to and that would definitely imply a different meaning that the models sucked and weren't developed correctly. But "even after" the models accommodated the outliers the results were bad. So G makes more sense.

That the comedian was so ____________ as to be unable to ____________ the effect she had on others was not lost on her audience, who quickly stood up to leave, hoping their action would at last ____________. Blank (i): A) coarse B) oblivious C) genteel Blank (ii): D) discern E) mitigate F) ignore Blank (iii): G) serve as an uncommon retort H) send an unambiguous message I) provide a cryptic counterpoint

Level: Hard -The key is to fill in the last blank; but I also had a strong intuition for what the blanks were going to be (gullible; figure out; clear warning) -A and E could seem like they are right but the TC doesn't give any clues that the comedian was "coarse" -The last blank is key since the complete opposite of that would have to go in the first blank which is "oblivious" -> the second blank follows to be "discern"

Many imagine philosophy appareled in a toga walking about the Greek agora, (i)______________ questions of great import; yet philosophy (ii) ______________ today, only we have traded the agora for the Internet: many online venues exist in which the intellectually curious discuss the very same questions that once reverberated through the open air of Athens' marketplaces. Blank (i): A) holding forth on B) disproving C) dismissing Blank (ii): D) continues to be imperilled E) is very much alive F) remains esoteric

Level: Hard -The second blank is easy to tackle first -The first blank was tough for me just because of the syntax but I knew it had to be related to the second blank and B and C did not make sense -Then I came up with my word "discussing" questions of great import; although I didn't know "holding forth on" is an idiom which means to debate I went with it

Ironically, for someone whose novels were populated with characters typically marked by a(n) (i)____________, the author himself made tabloid news with exploits that suggested not even a (ii) ____________. Blank (i): A) consummate urbanity B) piercing intellect C) unmitigated temerity Blank (ii): D) modicum of civility E) semblance of normalcy F) trace of wherewithal

Level: Hard -This is a double shift since both the blanks are similar -Ironically is used to show that something is "opposite" of what is assumed to be the case

A school of conservationist thought that continues to gain traction in academic circles contends that despite the most noble of intentions, the U.S. National Parks and Forests Services has, in allowing for the uncurbed growth of trees within parks, contributed to the ____________ forest fires. While it is true that park rangers can respond to fires quickly, often such fires are far fiercer than in areas not so ____________ trees and underbrush. That is not to say that all fires are deleterious: indeed forest fires play an appreciable role in the functioning of the ecosystem; they ____________ the growth of trees so that any given area is less likely to become densely wooded. Blank (i): A) decrease in B) depletion of C) outbreak of Blank (ii): D) choked with E) devoid of F) distant from Blank (iii): G) check H) foster I) eradicate

Level: Hard -This is tough to get through just because it is so long, it's almost a short reading passage -The clue for the first blank was "despite the most noble of intentions" meaning that the first blank has to convey the opposite of something good happening -The second blank is a syntactical nightmare for figure out but it's basically saying fire are much fiercer in areas that don't have "so many trees" so D makes sense -The last point shifts in tone and suggests that the forest fires could be good in that they keep the growth contained and so "check" makes the most sense

Claire's moods shifted drastically and unexpectedly - one moment she was ____________, chatting lively, the next she was ____________, looking forlornly out of the window. Blank (i): A) prolific B) dysphoric C) ebullient Blank (ii) D) unflappable E) taciturn F) despondent

Level: Hard -This isn't too bad but highlights a a good few points: 1) The point of hard GRE isn't to use tough words but force you to use clues and logic. I didn't know what dysphoric (feeling of distress) meant but I knew the second blank had to be "F" because of the clue "forlornly" thus GRE wants exact opposite so, ebullient means "joyously unrestrained" made the most sense

There is nothing more ____________ for first time writers to see that their cherished ideas are actually far less ____________ than they had imagined. Often a publisher, or even a friend, will point out that another writer already captured the same plot twist, or created an almost identical fictional world. This feeling stings even greater when the publication of the neophyte writer's work ____________ that of the more popular author; apparently the public often erroneously believes that the lesser known writer's work is derivative. Blank (i): A) galling B) bracing C) alluring Blank (ii): D) novel E) rigorous F) precedented Blank (iii): G) belittles H) precedes I) elaborates upon

Level: Hard -This one is just about picking up on clues that are important -The first blank I was stuck between bracing and galling but went with what I knew; bracing means to be invigorating and freshening and galling has the negative connotation I was looking for -The second blank is pretty straightforward especially after the sentence which follows it -The last blank can be tricky but we see the word "erroneously", which means that something here is being interpreted and it's about how an author's work might be attributed to someone else

The author's name in shining lights—on the book jacket, that is—____________ the collective nature of the enterprise: before hundreds of seasoned eyes even pore over a final copy, the author has had many expert readers help fashion the novel from its very inception. A) undermines B) belies C) informs D) reaffirms E) disambiguates

Level: Hard -This one is not so tough but the clue is subtle -It's talking about how "one" author's name shines on the front of the book but there are "numerous people" who help to get the book to the shelf -So the answer that best fits this is "belies"

For Nancy, teatime was no mere ______________: she was so persnickety that each step of the process—heating, steeping, and stirring—was executed with the solemnity and exactitude of a surgeon. A) respite B) duress C) vocation D) ceremony E) gambit

Level: Hard -This took a while just because of "cognitive dissonance" but the clue is that this person is very particular about teatime and it is no "joke" to her -So the opposite of being serious and demanding and particular would be "respite" -I did this via POE and found that none of the other words worked

The author, mocked by many for his simple, almost childlike prose, can at least not be begrudged the distinction of writing with _________. A) geniality B) naivety C) gusto D) anonymity E) lucidness

Level: Very Hard -Here I picked E because none of the other choices made much sense and also E preserved the sentence and went with the general meaning -The clue if I were confused and had to look for something explicit would be "simple" and "almost childlike" is elaborating on it but isn't the main point since it is a parenthetical phrase, which you should avoid if you can get the meaning without it

She was not so (i) ______________ as to begrudge the mathematician the fanfare he received after purportedly solving a hitherto intractable problem in number theory; nevertheless, once the furor died down she was not (ii) ______________ in pointing out what she believed to be some notable inconsistencies in his proof. Blank (i): A) savvy B) self-effacing C) churlish Blank (ii): D) loath E) charitable F) unstinting

Level: Very Hard -Here the first blank is easy to figure out since nothing besides "churlish" makes sense -The second blank is tough because there are a few choices that can work but ultimately this is what I did: 1) I went with unstinting first because it can mean two things (1- very generous or 2-unsparing) but then I ultimately decided to stick with the GRE definition of the word which is "very generous", also "unsparing" looking back now is a little too negative it would be better associated with a "screed" or "polemic" rather than pointing out mistakes 2) So then I was between charitable and loath and decided that "charitable" has a different connotation and is better associated with circumstances of being nice and giving and noticed that if I do that "charitable" and "unstinting" are too close in meaning and so neither can be the answer 3) So I went with "loath" which means "reluctant" and nothing about it seemed wrong and it was comprehensive and so the other two must be wrong

____________, she suddenly became ____________ , even conspiratorial, as the detectives, who had been stymied and had all but given up on extracting an iota of evidence from her, took sedulous notes. Blank (i): A) Unbidden B) Aghast C) Surprised Blank (ii): D) sullen E) contentious F) forthcoming

Level: Very Hard -I first interpreted the sentence wrong, thinking of "conspiratorial" and "sedulous" (which I confused with impetuous)... So I thought the answer was "contentious" -I picked the first blank correctly since it is the only thing that works (aghast and surprised are too close in meaning anyways) -For the second blank I came back and re-read to focus on "all but given up on extracting (even a small amount of evidence)... took sedulous notes". And realized that I'm not getting the meaning of sedulous correctly, which means to be "diligent". So basically the breakdown becomes that this person was a "hindrance" but suddenly started giving up information because the detectives started taking detailed notes

As spurious sightings of imaginary creatures that have captured the popular mind (i) ______________, however (ii) ________________ a story may be, once it has been circulated enough times, it will gather a patina of (iii) ______________. Blank (i) A) diminish B) entail C) suggest Blank (ii) D) clever E) apocryphal F) captivating Blank (iii) G) neglect H) truth I) deceit

Level: Very Hard -Not tough to figure out the last two but what makes this challenging is to realize that the word "however" is not a shifter -The word for the first blank has to be "suggest" and plugging it in makes the most sense; the only other word that could work is "diminish" but then the logic doesn't make sense. If the sightings of "false" things diminish then then people wouldn't take about them

All too often scientists are quick to ____________ findings that ostensibly fail to mesh with their own research; nonetheless, such a response is ____________ compared to the ____________, if not downright contemptuous, attitude they take towards a theory that questions the very foundation upon which their work rests. Blank (i) A) discuss B) doubt C) clutch at Blank (ii) D) unquestionably vitriolic E) positively muted F) slightly undiplomatic Blank (iii) G) complacent H) convivial I) dismissive

Level: Very Hard -Overall this was not too challenging to figure out -I started with the last blank and picked I which made the most sense with "contemptuous". I then knew that blank ii would contradict the third in some way and so E made the most sense -The first blank is ultimately dependent on the second and third and is still negative but is "muted" or "less contemptuous" than "dismissive". The best choice is "doubt"

Jansen's writing strikes many as (i) ______________ : for one who is capable of enduing even the most recondite topics with a(n) (ii) ______________ tone, his prose becomes (iii) ______________ in the informal correspondences he had with his contemporaries. Blank (i) A) pedantic B) forbidding C) paradoxical Blank (ii) D) acerbic E) cautious F) breezy Blank (iii) G) curiously stilted H) fully realized I) somewhat unguarded

Level: Very Hard -The first blank is surprisingly the easiest to figure out once you realize that the last two blanks are opposites each other -- so "paradoxical" works for the first blank -The clue for the other two blanks is "recondite", which means hard to understand; intractable -Keeping that in mind the only word that works best for the second blank is "breezy", there is no hint of "acerbic" or "cautious" -If that's the case then the last blank has to be opposite to "breezy" and "clumsy" or "stilted" works best

While society may regard science as some ____________ activity closed off to the ____________ masses, the daily life of a scientist--driving to work each day, checking emails, meeting deadlines--can seem ____________. Blank (i) A) grand B) arcane C) illicit Blank (ii) D) disheveled E) benighted F) huddled Blank (iii) G) irredeemably prosaic H) surprisingly quotidian I) relentlessly hectic

Level: Very Hard -The first blank was easy to figure out since the word that I came up with was "esoteric" and "arcane" is very close to that -The second blank I knew had to reflect that the masses are not "privy" to the activities of the scientist and settled with "benighted" which means a state of ignorance (lacking knowledge). It's also the only word that made sense -The last blank I had to be pick between G and H and G was too strong with the word "irredeemable" meaning you can't redeem yourself and that doesn't make much sense so H worked well -A clue that I didn't notice but could help for the first blank is "closed off", which is an explicit call for "arcane"

*Strategy: When you come up with a word for one blank and the other blank is in the same sentence, try to come up with a word for that blank that relates to the one you have figured out.* According to Lackmuller's latest screed, published under the title, Why We Can't Win at Their Game, special interest groups not nominally tied to ecological concerns have become so (i) ___________ the process of environmental policymaking that those groups who actually aim to ensure that corporate profit does not trump environmental health have been effectively (ii) __________. Lackmuller's contention, however, is (iii) __________ in that it fails to account for the signal achievements environmental groups have effected over the last 20 years—often to the chagrin of big business. Blank (i) A) marginalized in B) indebted to C) influential in Blank (ii) D) vindicated E) squelched F) lionized Blank (iii) G) somewhat tentative H) rarely myopic I) highly misleading

Level: Very Hard -This was tough just because there is so much verbiage it's hard to keep track of what the main point is and if you are on the right track. This is the process that I went through: -My ultimate clue that I was on the right track was the clue "screed" which is synonymous with "tirade" but it really is associated with expressing discontentment. So this author is feeling negative towards "special interest groups". -I filled the second blank first and the word I had come up with was "shut out" or "made ineffective" and the word that came closest to that was "lionized" the other two didn't make much sense -Then I looked for a word for the second blank which reflected an action that would "hinder" the environmentalists and went with "influential in" -The last blank is a shifter which suggests that the author is overlooking something and so H couldn't work, G doesn't make much sense since being tentative means being unsure or not permanent and so "highly misleading" is the best option

The number of speeding tickets one receives is by no means a reliable measure of ____________. Some ____________ drivers, in fact, prove that in certain cases the inverse is true. That is those savvy enough to have availed themselves of the latest cellular phone applications receive up-to-the-minute information on the presence of highway patrolmen—greater excess speed, in these instances, simply implies a greater ____________. Blank (i) A) awareness B) culpability C) susceptibility Blank (ii) D) affluent E) intrepid F) resourceful Blank (iii) G) degree of confidence H) sense of vulnerability I) likelihood of entrapment

Level: Very Hard -Took me a long time to feel certain but I had the right answers picked all along -The key is to break down the sentence into something you can understand -The second blank is easiest to solve since it has the most clue so it is F -The first blank is not that bad either, "blame" makes the most sense -The last blank is a bit tougher but you have to understand the sentence to feel comfortable... It is basically saying "greater speed simple implies greater ______" and G makes the most sense since there is no clue for H since they wouldn't be vulnerable since they have these gadgets and I doesn't make sense either

Mulcahy, in averring that most literary criticism has become so filled with abstruse jargon as to be practically indecipherable to anyone save its practitioners, is himself (i) ___________: his main point will be discernible only to the very community he seeks to (ii) ___________. Blank (i): A) uncertain B) complicit C) enlightened Blank (ii): D) defend E) impugn F) inform

Level: hard -I didn't know what aver meant but ignored it (it means to assert to be the case) then I picked B (since he himself has become indecipherable) -Then if that is the case D makes the most sense


Kaugnay na mga set ng pag-aaral

FAR - F2 Financial Reporting and Disclosures

View Set

Fundamentals of Nursing (Chap 16 Documenting, Reporting, Conferring, and using Informatics)

View Set

ENT chapter 7 (head and neck surgery)

View Set

Chapter 3: Genetics Concept Questions

View Set